You are on page 1of 56

USMLE WORLD STEP 2 CK (INTERNAL MEDICINE) *Block

2*

BLOCK # 2
47 Internal Medicine Cardiology 70 Internal Medicine Miscellaneous
48 Internal Medicine Neurology 71 Internal Medicine Ophthalmoloqy
49 Internal Medicine Respiratory 72 Internal Medicine Respiratory
50 Internal Medicine Cardiology 73 Internal Medicine Neurology
51 Internal Medicine Biostatistics 74 Internal Medicine ENT
52 Internal Medicine Rheumatology 75 Internal Medicine GIT
53 Internal Medicine Neurology 76 Internal Medicine ID
54 Internal Medicine Respiratory 77 Internal Medicine Electrolytes
55 Internal Medicine ID 78 Internal Medicine Hem&Onc
56 Internal Medicine Ophthalmology 79 Internal Medicine Endocrinology
57 Internal Medicine Hem&Onc 80 Internal Medicine Respiratory
58 Internal Medicine Hem&Onc 81 Internal Medicine Respiratory
59 Internal Medicine Poisoning 82 Internal Medicine Hepatology
60 Internal Medicine Hepatology 83 Internal Medicine ENT
61 Internal Medicine Neurology 84 Internal Medicine ID
62 Internal Medicine Biostatistics 85 Internal Medicine Biostatistics
63 Internal Medicine Miscellaneous 86 Internal Medicine Cardiology
64 Internal Medicine Dermatology 87 Internal Medicine Hem&Onc
65 Internal Medicine GIT 88 Internal Medicine Genitourinary
66 Internal Medicine ID 89 Internal Medicine Neurology
67 Internal Medicine Electrolytes 90 Internal Medicine Biostatistics
68 Internal Medicine Hepatoloqy 91 Internal Medicine ID
69 Internal Medicine GIT 92 Internal Medicine Hem&Onc

__________________________________________________________ 53
USMLE WORLD STEP 2 CK (INTERNAL MEDICINE) *Block
2*
Q NO 47: A 56-year-old white male is recovering well from his acute A.
anterior wall Ml that was treated with thrombolytic therapy 4 days
ago. On day 4 of his hospitalization, he develops new chest pain that
is similar to his previous pain of myocardial infarction in severity,
radiation and character. The pain is not relieved by sublingual
nitroglycerin. He is hemodynamically stable. His lung fields are clear
and heart sounds are normal. EKG shows hyperacute/waves and new
ST segment elevation in leads V1, V2, and V3. He is suspected to
have re-infarcted. Which of the following biomarkers of cardiac injury
can establish the diagnosis of re-infarction in this setting?
Cardiac troponin I
B. Cardiac troponin I
C. LDH
D. CR-MB
E. Myoglobin

Explanation:
Explanation: CK-MB fraction has a high specificity for an acute MI
(slightly lower than cardiac troponins). It begins to rise within 4-6
hours after Ml and returns to baseline within 48-72 hours. Its high
specificity and rapid return to the baseline makes it the biomarker of
choice for the diagnosis of a recurrent MI.
Cardiac troponins I and cardiac troponins I are proteins that control
the interaction of actin and myosin. They are more specific than all
the other biomarkers of cardiac injury being used for the diagnosis of
MI. They begin to rise 4-6 hours after an MI, and remain elevated for
10 days. They have now become the primary biochemical tests used for
the diagnosis of acute MI. They have also replaced LDH for the
retrospective diagnosis of MI. But because of their persistent
elevation for 10 days after an MI, they cant be used to establish the
diagnosis of re-infarction within 1-2 weeks after an MI.
LDH is present in cardiac muscle and other tissues like red cells,
kidney, liver, and skeletal muscle. It begins to rise within 4-6 hours,
reaches its peak in 24-48 hours and remains elevated for 6-7 days.
Because of its low specificity and persistent elevation, it is not
useful for the diagnosis of a re-infarction. It has also been replaced
by cardiac troponins for the retrospective diagnosis of MIs as it lacks
specificity.
Myoglobin is a heme-protein found in many tissues. It has a rapid rise
and return to the baseline after an acute Ml. It can be used to detect
recurrent injury but because of lack of specificity, CK-MB is a better
choice.

Educational Objective:
Emphasize the importance of CK-MB for the diagnosis of recurrent
myocardial infarction.

54 __________________________________________________________
USMLE WORLD STEP 2 CK (INTERNAL MEDICINE) *Block
2*
Q NO 48: A 26-year-old man comes to the emergency department because
he is suffering from the worst headache of his life. He feels nauseated and
is photosensitive. His blood pressure is 160/90mm Hg. heart rate is 83/min,
and temperature is 36.5C (97.7). The physical examination reveals no focal
neurological symptoms, except for some meningismus and vertigo, which is
not localized to either side. CSF examination reveals the presence of
xanthochromia. What is the major cause of morbidity and mortality in a
patient with the above condition?

A. Post-angiographic complications
B. Vasospasm with symptomatic ischemia and infarction
C. Secondary infection
D. Post-surgical complications
E. Nimodipine use
F. Cardiorespiratory arrest from increased CSF pressure

Explanation:
This patient has a subarachnoid hemorrhage, and is at risk for
subsequent vasospasm of the arteries at the base of the brain.
Vasospasm following SAH occur in approximately 30% of patients, and is
the major cause of morbidity and mortality in such patients. In
affected patients, the signs of ischemia usually appear in about 7 days
after the SAH.
(Choice E) Calcium channel blockers (e.g., nimodipine) are used to
prevent vasospasm in patients with SAH.
(Choices A, C, and D) Secondary infections, as well as complications
after angiography and surgery, are not as prevalent as vasospasm
following SAH.

Educational Objective:
Vasospasm is the major cause of morbidity and mortality in patients
with subarachnoid hemorrhage (SAH). Calcium channel blockers (e.g.,
nimodipine) are used to prevent vasospasm in patients with SAH.

__________________________________________________________ 55
USMLE WORLD STEP 2 CK (INTERNAL MEDICINE) *Block
2*
Q NO 49: A 66-year-old man is admitted to the hospital after he developed
severe breathlessness. He was in his usual state of health 3 days ago when
he developed fever with rigors and productive cough. The sputum is
greenish in color and smells bad. He reports 100 pack-year history of
smoking. He was started on azithromycin along with the supplemental
oxygen, ipratropium inhaler, albuterol inhaler and IV meth prednisolone. On
physical examination, his pulse is 96/min, Blood Pressure 100/60mm Hg:
Temperature is 102 F and Respiratory Rate is 24/min. His lung examination
reveals rales all over the chest along with occasional rhonchi. Expiratory
phase is prolonged. EKG reveals multifocal atrial tachycardia. Which of the
following agents will worsen the arrhythmia?

A. Azithromycin
B. Oxygen
C. Ipratropium inhalation
D. Albuterol inhalation
E. Methyl prednisolone

Explanation:
Explanation: The patient has multifocal atrial tachycardia, which is
associated with the exacerbation of chronic obstructive pulmonary
disease (COPD). This patient needs measures to reduce the severity of
hypoxemia. The therapy for this chaotic atrial tachycardia is directed
primarily against the underlying disease. However, some agents like
theophylline and beta agonists may actually worsen the arrhythmia. So
they may be used judiciously. The patient should be evaluated for any
underlying electrolyte imbalance. Remember beta agonists also have the
tendency to lower the potassium levels, which have further arrhythmic
effects. Azithromycin, Ipratropium and steroids have no effects on the
arrhythmia. Oxygen will help to stop the arrhythmia.

Educational Objective:
Know how to manage various clinical situations in exacerbation of
chronic obstructive pulmonary disease.

56 __________________________________________________________
USMLE WORLD STEP 2 CK (INTERNAL MEDICINE) *Block
2*
Q NO 50: A 40-year old female immigrant from Asia comes for evaluation of
her dyspnea. Her dyspnea was mild 6 months ago but it has now
progressed to dyspnea, even at rest. She denies any chest pain, syncope, or
palpitations. She is a non-smoker and non-alcoholic. Her past medical
history is significant for pulmonary tuberculosis. Her PR: 82/min: BP:
135/70 mmHg; Temperature: 37.1C (98.8F); RR: 14/min. On her
examination, significant physical findings are jugular venous distension,
bilateral ankle edema, and tender hepatomegaly. Chest x-ray shows
pericardial calcifications. Which of the following set of physical findings is
most likely to be present in this patient?

A. Early third heart sound and inspiratory increase in jugular venous


pulse
B. Water hammer pulse and pistol shot femorals
C. Tapping apex beat and malar flush
D. Pulsus paradoxus and hypotension
F. Pansystolic murmur at left lower sternal border

Explanation:
This patient, most likely, has constrictive pericarditis. The finding
of pericardial calcifications on the chest X-ray is an important clue.
The etiology of pericarditis is probably from her prior history of
Tuberculosis. The early third heart sound, that is also called pen
cardial knock and the inspiratory increase in the jugular venous
pressure (Kussmauls sign), are important physical findings of
constrictive pericarditis. Kussmauls sign is also present in right
sided heart failure, severe tricuspid regurgitation, right ventricular
infarction and cardiac tamponade.
Water hammer or collapsing pulse and pistol shot femoral pulses are
diagnostic clues to aortic regurgitation. These physical findings occur
due to a hyperdynamic circulation and early diastolic runoff of aortic
insufficiency.
Tapping apex beat and malar flush are important physical findings of
mitral stenosis. Pulsus paradoxus is defined as greater than 10-mmHg
fall of the systolic blood pressure during inspiration.
Pulsus paradoxus and hypotension point toward the diagnosis of
pericardial tamponade. The former is uncommon in constrictive
pericarditis unless an effusion is present. It may also be present in
severe airway obstruction and superior vena cava obstruction.
A pansystolic murmur at the left sternal border is usually seen in
tricuspid regurgitation.

Educational Objective:
Know the characteristic clinical findings of constrictive pericarditis.

__________________________________________________________ 57
USMLE WORLD STEP 2 CK (INTERNAL MEDICINE) *Block
2*
Q NO 51: A group of investigators are planning a study to evaluate the
relationship between serum fibrinogen level and the incidence of acute
coronary syndrome. They assume that serum fibrinogen level is a
normally distributed variable in the population of interest. Which of the
following statements is most consistent with this assumption?

A. Mean is greater than median


B. Mean is greater than mode
C. Mean is equal to median
D. Median is greater than mean
E. Mode is greater than mean

Explanation:
The normal distribution is one of the most popular statistical
distributions. Interestingly, many variables in real life (e.g.,
laboratory values) are normally distributed or close to normal. The
normal distribution has some nice statistical properties, and is easy
to work with. It is bell-shaped and symmetric. Consequently, all its
measures of central tendency are equal: mean = median = mode (In real
life, the values are very close to each other). Skewed distributions do
not have this property.
(Choices A and B) In a positively skewed distribution (tail on the
right), the mean is greater than the median and greater than the mode.
(Choices D and E) In a negatively skewed distribution (tail on the
left), the mean is less than the median and less than the mode.

Educational Objective:
A normal distribution is symmetric and bell shaped. All its measures of
central tendency are equal: mean = median = mode.

58 __________________________________________________________
USMLE WORLD STEP 2 CK (INTERNAL MEDICINE) *Block
2*
Q NO 52: A 65-year-old white male presents with severe pain and A. X
weakness of the right shoulder. His pain is worsened when he tries ray
to position his arm above the shoulder level or when he pulls or
pushes. He also complains of night pain when he lies over the
affected shoulder. He is unable to reach overhead and cant lift
anything with an outstretched arm. He has a 40 pack-year history
of cigarette smoking and is a chronic alcoholic. He has no other
significant medical problem. His father died of multiple myeloma.
His vitals are stable and he is afebrile. On examination, there is
limitation of mid arc abduction and external rotation that does not
improve after lidocaine injection. There is no swelling, redness or
warmth. His neurovascular status is intact. What is the most
appropriate next step to diagnose his problem?
shoulder
B. Chest-X-ray
C. Serum immunoelectrophoresis
D. MRI of the shoulder
E. Aspiration of the joint
F. Bone scan

Explanation:
Explanation: This patient most likely is suffering from rotator cuff
tear which presents with shoulder pain aggravated by movements like
pushing, pulling and positioning the arm above the shoulder as well as
weakness of shoulder resulting in functional impairment. Examination
may show limitation of mid arc abduction or external rotation or both
which does not reverse with lidocaine injection thus differentiating
the rotator cuff tear from rotator cuff tendinitis.
Definitive diagnosis of rotator cuff tear requires MRI or arthrography.
MRI should be performed when rotator cuff tear is suspected on clinical
grounds and patient has no other serious medical problem that will make
the corrective surgery difficult. The above patient does not have any
serious medical problem and his symptoms are incapacitating and he is
very likely to be benefited from surgery.
Aspiration of the joint is indicated when there is suspicion for septic
or crystal induced arthritis. The above patient has no swelling, warmth
or redness of his joint and septic arthritis, gout and pseudogout are
highly unlikely in this patient.
Though this patient has family history of multiple myeloma, his
clinical picture is not compatible with the diagnosis of multiple
myeloma and there is no need for bone scan or serum
immunoelectrophoresis.
The only problem that this patient has is shoulder pain and weakness
that is most likely due to rotator cuff tear and chest x-ray is not
going to be helpful in making the diagnosis. Rather plain x-ray of the
shoulder not plain chest x-ray is recommended when there is suspicion
of shoulder pathology. Obliteration or narrowing of acromiohumeral
space will be seen in complete tears. Calcification may also be seen in
chronic cases.

Educational Objective:
MRI is the definitive diagnostic study for rotator cuff tear.

__________________________________________________________ 59
USMLE WORLD STEP 2 CK (INTERNAL MEDICINE) *Block
2*
Q NO 53: A 23-year-old white man is brought to the emergency A.
department (ED) by an ambulance due to an epileptic seizure.
He fell on the sidewalk while going home from a pub, where he
had two beers. A witness noted tonic-clonic movements of all
four extremities for about one minute. This seizure was his first
episode, and lasted 30 minutes. In the ED, he is in a state of
partial confusion and disoriented to time, place and person. The
physical examination does not reveal any focal neurologic
pathology. His eye exam does not show any papilledema. His
airway is secured, and his breathing is normal. CBC, serum
electrolytes, EKG and chest x-ray are normal. Urine toxicology
screen is ordered, and lorazepam is given. What is the most
appropriate next step in the management of this patient?
Psychiatric consultation
B. Brain computed tomography without contrast
C. Brain computed tomography with contrast
D. Lumbar puncture
E. Electroencephalogram

Explanation:
In a patient who presents with focal neurologic signs and symptoms and
an unclear history, itis essential to rapidly exclude life-threatening
processes such as an intracranial hemorrhage; therefore, the most
appropriate initial test in this case is a CT scan without contrast.
Further management is dependent on the CT findings. In addition, this
patient may be loaded with Dilantin to suppress or prevent seizure
activity.
(Choice B) CT with contrast is usually indicated if brain tumors or
other mass lesions (toxoplasmosis, lymphoma, cancer) is the suspected
cause of the patients symptoms. If the non contrast CT excludes the
hemorrhage, then MR or CT with contrast of the brain is the next step.
(Choice D) A lumbar puncture is not indicated in an afebrile patient
with no meningeal signs.
(Choice E) EEG is the gold standard for the documentation of
epileptiform activity. However, this is not the next step.
(Choice A) Psychiatric consultation is usually done when there is
evidence of drug intoxication.

Educational Objective:
CT scan of the head without contrast is the initial diagnostic test of
choice when a patient presents with impaired consciousness or seizures
or focal neurologic signs.

60 __________________________________________________________
USMLE WORLD STEP 2 CK (INTERNAL MEDICINE) *Block
2*
Q NO 54: A 54-year-old male undergoes a complex abdominal operation.
Two days later he becomes short of breath and tachypneic. He requires
mechanical ventilation. His chest x-ray reveals bilateral fluffy infiltrates.
Hemodynamics reveal that the patient is not in congestive heart failure,
repeated sputum cultures are negative and his V/Q scan is of a low
probability. His WBC count is 10,000/cmm with 1% bands. The patient
continues to be difficult to oxygenate. The patient most likely has:

A. Pneumonia
B. Atelectasis
C. ARDS
D. Pulmonary embolism
E. Pneumothorax

Explanation:
In a postoperative patient who presents with dyspnea and tachypnea, one
has to exclude Ml, PE, pneumonia and ARDS. In the above patient, there
is no evidence of heart failure and all the sputum cultures are
negative. In addition his V/Q scan is negative so the probability of a
PE is low. In the presence of bilateral fluffy infiltrates and hypoxia
with a difficulty in oxygenation, the diagnosis of ARDS must be
entertained. The earliest sign is often tachypnea followed by dyspnea.
The physical exam and chest x-ray are usually normal initially.
Choice A: Pneumonia can definitely present in a postoperative patient.
However, in the above patient, the sputum cultures are negative.
Pneumonia generally occurs from three-to-seven days alter surgery.
Sometimes if the sputum cultures are negative and suspicion is high for
pneumonia, bronchoscopic cultures are obtained. The patient with
pneumonia may have excessive secretions, fever and an elevated WBC.
Choice B: Atelectasis is collapse of segments of lung. This can lead to
dyspnea and tachypnea, depending on the degree of atelectasis.
Postoperative atelectasis is a major cause of fever. It may be due to
mucus plugs, postoperative pain, decreased mobility and poor pulmonary
toilet. The diagnosis is made on a chest x-ray.
Choice D: A VJQ scan reveals that there is a low probability of a PE.
It is very unlikely to have a PE with low probability V/Q scan. A PE
generally presents from five-to-seven days after surgery. Despite
prophylactic measures to prevent it, it still causes thousands of
deaths in North America each year.
Choice E: A pneumothorax can cause dyspnea and tachypnea. In this
patient, the chest x-ray does not reveal any evidence of a
pneumothorax. A pneumothorax on a chest x-ray will be easily
recognized. Small pneumothoraces do not generally cause symptoms.

Educational objective:
ARDS can present with dyspnea, tachypnea and bilateral fluffy
infiltrates on a chest x-ray. It is a condition where hypoxemia
persists and it becomes difficult to oxygenate the patient.

__________________________________________________________ 61
USMLE WORLD STEP 2 CK (INTERNAL MEDICINE) *Block
2*
Q NO 55: A 73-year-old diabetic man presents with low-grade fever, facial
pain over his right maxilla, and bloody nasal discharge for the last three
days. For the last day, he has had diplopia. He was diagnosed with
diabetes mellitus 10 years ago. For the last year, he has been on insulin.
His most recent hemoglobin A1C was 12.0. His temperature is 39.O C
(102.2 F), pulse is 88/min, and blood pressure is 130/76mm Hg.
Examination shows right-sided nasal congestion and necrosis of the right
nasal turbinate with tenderness over the right maxillary sinus. There is
chemosis and proptosis of his right eye. CT scan shows opacification of the
right maxillary sinus. Which of the following is the most likely causative
organism?

A. Rhizopus species
B. Staphylococcus aureus
C. Pseudomonas aeruginosa
D. Haemophilus influenzae
E. Moraxella catarrhalis
F. Cryptococcus neoformans
G. Candida albicans

Explanation:
This patient is most likely suffering from mucormycosis of the nose and
maxillary sinus. The most common etiologic agent is Rhizopus. Poorly
controlled diabetes mellitus predisposes to this disease. Low-grade
fever, bloody nasal discharge, nasal congestion, and involvement of the
eye with chemosis, proptosis, and diplopia are important features.
Involved turbinates often become necrotic. Invasion of local tissues
can lead to blindness, cavernous sinus thrombosis, and coma. If left
untreated, mucormycosis can lead to death in days to weeks.
(Choice B) Staphylococcus aureus is a common cause of cellulitis.
(Choice C) Pseudomonas is a cause of malignant otitis externa in
diabetic patients; it may cause black necrotic lesions in the ear.
(Choices D and E) H. Influenza and Moraxella catarrhalis are common
causes of bacterial sinusitis. These usually do not cause necrotic
infections.
(Choice F) Cryptococcus neoformans causes meningitis in
immunocompromised patients.
(Choice C) Candida can cause thrush and vaginitis in diabetic patients.
Necrotic lesions are uncommon.

Educational Objective:
The association between diabetes mellitus and mucormycosis is
frequently tested on the USMLE. The most common cause of mucormycosis
is Rhizopus.

62 __________________________________________________________
USMLE WORLD STEP 2 CK (INTERNAL MEDICINE) *Block
2*
Q NO 56: A 60-year-old woman comes to the emergency department due
to a sudden onset of severe pain in her left eye with blurred vision,
nausea, and vomiting. The symptoms began a few minutes ago, while she
was watching a movie in a near by theater. Her blood pressure is 140/90
mm Hg, pulse is 82/min, respirations are 14/min, and temperature is 98.4
F. Examination reveals decreased visual acuity. Her left eye appears red,
with a hazy cornea, shallow anterior chamber, and dilated, fixed pupil. Her
left eye is stony hard to touch. What is the most likely diagnosis?

A. Primary open angle glaucoma


B. Conjunctivitis
C. Acute angle closure glaucoma
D. Anterior uveitis
E. Corneal abrasion

Explanation:
Angle closure glaucoma occurs with closure of a pre-existing narrow
anterior chamber angle. It predominantly occurs in people aged 55-70
years. It presents with an acute onset of severe eye pain and blurred
vision associated with nausea and vomiting. It usually occurs following
pupillary dilation, which may occur in darkened movie theaters, during
times of stress, or due to drug intake. Examination reveals a red eye
with steamy cornea and moderately dilated pupil that is non reactive to
light. The anterior chamber is shallow with inflammatory changes.
Tonometry reveals increased intraocular pressure. Intravenous
acetazolamide (with subsequent oral administration) may lower the
intraocular pressure. Permanent cure is offered with laser peripheral
iridotomy.
(Choice A) Open angle glaucoma has an insidious onset, with gradual
loss of peripheral vision resulting in tunnel vision. Other
characteristic features are persistently increased intraocular pressure
and pathologic cupping of the optic disc.
(Choice B) Conjunctivitis is characterized by very mild pain. The
cornea is clear. Pupilla size and response to light is normal. Visual
acuity is not affected.
(Choice D) Uveitis presents with moderate pain and blurred vision.
Cornea may be hazy. The anterior chamber shows flare and cells on slit
lamp examination. The pupil is constricted with a poor light response
(In acute glaucoma, the pupil is dilated and is nonreactive to light).
(Choice E) Corneal abrasion presents with severe pain and photophobia.
There is usually a history of trauma to the eye. Slit lamp examination
with fluorescein will reveal the corneal abrasion.

Educational Objective:
Angle closure glaucoma occurs predominantly in people aged 55-70 years.
It presents with an acute onset of severe eye pain and blurred vision
associated with nausea and vomiting. Examination reveals a red eye with
steamy cornea and moderately dilated pupil that is non reactive to
light.

__________________________________________________________ 63
USMLE WORLD STEP 2 CK (INTERNAL MEDICINE) *Block
2*
Q NO 57: A 35-year-old male is brought to the emergency room with
headaches and confusion for the past 2 days. He denies any focal
weakness or sensory symptoms. His past medical history is significant
for HIV and hepatitis C infections for which he is not receiving
therapy. The remainder of his medical history is unobtainable due to
his mental status. On physical exam, he has a temperature of 37.9C
(100.2F), a blood pressure of 140/86mm Hg, a pulse of 96/min, and
respirations of 16/min. Mild icterus is present. The patients
oropharynx is clear and his neck is supple and without rigidity.
Examination of his chest and abdomen are unremarkable. Neurologic
examination reveals no focal deficits. Laboratory studies show:
Complete blood count
Hemoglobin 7.6 g/L
MCV 85 fl
Reticulocytes 8.1%
Platelet count 80,000/mm
Leukocyte count 3,500/mm
Chemistry panel
Blood urea nitrogen (BUN) 30 mg/dL
Serum creatinine 2.2 mg/dL
Serum calcium 10.0 mg/dL
Blood glucose 98 mg/dL
Liver studies
Total bilirubin 3.6 mg/dL
Direct bilirubin 1.0 mg/dL
Alkaline phosphatase 120 U/L
Aspartate aminotransferase (SGOT) 178 U/L
Alanine aminotransferase (SGPT) 255 U/L
Which of the following is the most appropriate next step in the
management of this patient?

A. Liver biopsy
B. Bone marrow biopsy
C. Right upper quadrant ultrasound
D. Lumbar puncture
E. CT scan of the head
F. Peripheral blood smear
G. Hemoglobin electrophoresis

Explanation:
This patient has several metabolic abnormalities requiring further
investigation. He has a normocytic (MCV = 85 fl) hemolytic anemia with
an appropriate bone marrow response (evidenced by the high reticulocyte
count), an increased indirect bilirubin level, thrombocytopenia and
renal failure. Together these findings suggest a possible diagnosis of
thrombotic thrombocytopenic purpura-hemolytic uremic syndrome (TTP-
HUS). Absence of fever was noted in a recent case series of patients
with TTP-HUS. However, the patient also has a history of HIV and
hepatitis C, both of which can cause thrombocytopenia and anemia of
chronic disease. Both can also cause renal failure (HIV focal
segmental glomerulosclerosis: HCV membranoproliferative
glomerulonephritis), and the patients AST and ALT elevations could be
due to chronic liver disease from HCV. A key clue to the diagnosis of
TTP-HUS in this patient is his reticulocytosis. A peripheral smear with
> 1% schistocytes would be virtually diagnostic of microangiopathic
hemolytic anemia (MAHA), a component of HP-HUB. However, MAHA can also

64 __________________________________________________________
USMLE WORLD STEP 2 CK (INTERNAL MEDICINE) *Block
2*
occur in DIG and malignant hypertension. Coagulation tests would help
to rule out DIG. (DIG patients tend to bleed and have abnormal
coagulation studies. HP-HUS patients do not bleed despite their low
platelet count.) Malignant hypertension should be considered if there
is a history of hypertension and evidence of hypertensive retinopathy
on funduscopic exam. Importantly, HIV patients are at increased risk
for HP.

Educational Objective:
Unexplained hemolytic anemia and thrombocytopenia in a patient with
renal failure and neurologic symptoms should raise strong suspicions
for TIP-HUS. HIV increases the risk for HP. Without prompt institution
of appropriate therapy, HP-HUS proves lethal in 80% of patients. A high
degree of clinical suspicion is thus required.

__________________________________________________________ 65
USMLE WORLD STEP 2 CK (INTERNAL MEDICINE) *Block
2*
Q NO 58: A 50-year-old woman comes to the office and complains of right
shoulder pain which radiates to her hand. She has had cough for many
weeks, and feels more tired than the usual. In addition, her fingers are
always swollen, and she now finds it difficult to walk because her knees
give way. She has had rheumatoid arthritis for the past 10 years and is
used to having pain in her joints: however, she believes that this pain is
not due to her arthritis. She takes celecoxib for rheumatoid arthritis. She
admits to smoking one pack of cigarettes daily for the past 25 years, and
to drinking one odd beer every night. All her family members have
crippling rheumatoid arthritis. Her vital signs are stable. She is afebrile.
Physical examination reveals drooping of the right eyelid and miosis. What
is the best next step in the management of this patient?

A. CT scan of head and neck


B. X-ray of the shoulder
C. Steroid therapy
D. Chest x-ray
E. Nerve conduction study
F. Electromyography

Explanation:
Until proven otherwise, suspect lung cancer in smokers who present with
Homers syndrome (i.e., partial ptosis, miosis, anhidrosis). Other
presentations of lung cancer can include any of the following: cough
with bloody sputum, shortness of breath, chest pain, loss of appetite,
and weight loss. Additional symptoms that may be associated with this
disease are: weakness, swallowing difficulty, hoarseness or changing
voice, facial swelling, facial paralysis and eyelid drooping. Most lung
cancers are caused by cigarette smoking. The more significant the
smoking history the greater the risk of lung cancer. Second-hand smoke
has also been shown to increase the risk. High levels of pollution,
radiation, and asbestos exposure may also increase the risk.
In this vignette, there are many clinical clues to the diagnosis of
lung cancer. In addition, the patients shoulder pain can be attributed
to the direct extension of the lung cancer, as well as involvement of
the nerves of the brachial plexus and sympathetic trunk. Simple chest
x-ray should be the first test in the evaluation of this patient.
(Choice C) Starting steroids is not really the best option. Although
the patient has a history of rheumatoid arthritis, her current
complaint of pain does not appear to be related to it. Furthermore, her
joint problems might be a part of hypertrophic osteoarthropathy.
(Choice D)There is no history of trauma; therefore, obtaining a
shoulder x-ray is not the best next step in this patients management.
(Choices E and F) This is not a nerve disease, so nerve conduction and
EMO should not be used in the management of this patient.

Educational Objective:
Until proven otherwise, suspect lung cancer in smokers who present with
Homers syndrome (i.e., partial ptosis, miosis, anhidrosis). Simple
chest x-ray should be the first test in patients with suspected lung
cancer.

66 __________________________________________________________
USMLE WORLD STEP 2 CK (INTERNAL MEDICINE) *Block
2*
Q NO 59: A 28-year-old man is brought to the emergency room because of
a black widow spider bite on his left forearm. He was cleaning his garage
while this happened. He has severe pain at the site of the bite, along with
generalized abdominal discomfort. He has no other medical problems. He
does not use tobacco, alcohol, or drugs. He has no known drug allergies.
His temperature is 37.50 (99 F), blood pressure is 110/68mm Hg, pulse is
88/min, and respirations are 16/min. He is very anxious to know the effect
of black widow spider bites. Which of the following is the most appropriate
next step in management?

A. Calcium gluconate
B. Dapsone
C. Penicillin
D. Rifampin
E. Steroids

Explanation:
Black widow spiders are usually found in warm parts of the USA. The
spider got the name due to the belief that the female spider kills the
male after mating. These spiders usually do not bite humans, unless
provoked. The toxin produced by the black widow affects the nervous
system. Not all bites lead to severe reactions. Some bites go
unnoticed. Other bites usually lead to severe pain at the site. After a
while, the bite mark shows signs of inflammation along with tremors,
muscle weakness, abdominal pain, nausea, and vomiting. The abdominal
pain may mimic pancreatitis or appendicitis. The abdomen may have
boardlike rigidity but is not tender to palpation.
Treatment includes the following steps:
Basic or advanced life support.
The site should be inspected and cleansed.
Tetanus immunization should be updated.
Hypertensive episodes related to the envenomation may require
treatment with nitroprusside.
* Abdominal cramps are best treated with calcium gluconate.
* Diazepam or dantrolene can be used to alleviate muscle spasms.
(Choice B) Because of its leukocyte inhibitory properties, dapsone is
used to reduce the extent of local necrosis in patients with brown
recluse spider bites.

Educational Objective:
1. Acute abdomen is a feature of black widow spider bite and is best
treated with a combination of calcium gluconate and muscle relaxants.
2. Brown recluse spider bites produce an extensive localized skin
necrosis resembling a pyoderma gangrenosum. Dapsone is used to reduce
the extent of local necrosis in patients who have been screened for
glucose-6-phosphate dehydrogenase (G6PD) deficiency.

__________________________________________________________ 67
USMLE WORLD STEP 2 CK (INTERNAL MEDICINE) *Block
2*
Q NO 60: A 22-year-old Caucasian male college student presents to his
primary care physician complaining of a two-day history of right upper
abdominal pain and not feeling well. He also admits to having
unusually dark urine, feeling fatigued and anorexic, and developing a
sudden aversion to cigarette smoking. He returned from a trip to Mexico
with his family approximately three weeks ago. His sister has similar
complaints but has not yet sought medical treatment. He is not taking
any medications and has no known drug allergies. He does not smoke
cigarettes and drinks beer primarily on social occasions. Physical
examination reveals yellow sclera, right upper quadrant tenderness, and
hepatomegaly. Total bilirubin is measured at 6.0 mg/dL. Which of the
following is the most likely cause of hepatitis in this patient?

A. Hepatitis D
B. Hepatitis B
C. Hepatitis C
D. Hepatitis A
E. Infectious mononucleosis

Explanation:
The clinical picture of viral hepatitis can be divided into three
phases: the prodromal phase, the icteric phase, and the convalescent
phase. Given this patients jaundice and worsening prodromal symptoms,
he is considered to be in the icteric phase of acute hepatitis.
Infection with the hepatitis A virus is strongly suggested by his
recent travel to an endemic country and his sisters similar clinical
complaints.
The pathogen responsible for hepatitis A is an RNA picornavirus with an
average incubation period of 30 days. Transmission occurs through the
fecal-oral route and is common in areas with overcrowding and poor
sanitation. Outbreaks frequently result from contaminated water or
food. Onset is acute, and symptoms can include malaise, fatigue,
anorexia, nausea, vomiting, mild abdominal pain, and an aversion to
smoking. Hepatomegaly is commonly seen. AST and ALT spike early in the
illness, followed by increases in bilirubin and alkaline phosphatase.
Fortunately, hepatitis A infection is a self-limiting disease and does
not progress to chronic hepatitis, cirrhosis, or hepatocellular
carcinoma. The mortality rate is less than 0.2%. although a
significantly prolonged prothrombin time correlates with increased
mortality.
Treatment of hepatitis A infection is largely supportive, with complete
recovery expected in 3-6 weeks. Close contacts of individuals with
hepatitis A should promptly be given immune globulin. People considered
at high risk (e.g., those living in or traveling to endemic areas,
those with chronic liver disease or clothing-factor disorders, men who
have sex with men) should be given the hepatitis A vaccine as
prophylaxis.
(Choice B) Hepatitis B is a DNA virus with an incubation period of 30-
180 days. Onset is typically insidious, and transmission can occur
sexually, parenterally, or vertically. Chronic hepatitis develops in 1-
2% of immunocompetent adults and 90% of newborns. These chronic
carriers of HBV are at significant risk of developing cirrhosis and

68 __________________________________________________________
USMLE WORLD STEP 2 CK (INTERNAL MEDICINE) *Block
2*
hepatocellular carcinoma. Recombinant vaccine and hepatitis B immune
globulin are the available preventative measures.
(Choice C) Hepatitis C is an RNA virus with an average incubation
period of 40-50 days. Clinical illness is mild and patients are often
asymptomatic. Infection with hepatitis C virus primarily occurs
parenterally, but sexual and vertical transmission are also possible.
Chronic hepatitis develops in >80% of patients, who are then at
increased risk of cirrhosis or hepatocellular carcinoma. No vaccine or
immunoglobulins are available, though treatment with interferon and
ribavirin usually slows disease progression. Clinically significant
acute hepatitis from hepatitis C infection is very rare.
(Choice A) Hepatitis D is an incomplete RNA virus that causes hepatitis
only in association with hepatitis B infection. It is transmitted by
percutaneous, sexual, and perinatal routes. Superinfection with HDV in
a patient with chronic hepatitis B can result in fulminant hepatitis or
severe chronic hepatitis that quickly progresses to cirrhosis.
Vaccination against hepatitis B is the best means of preventing
infection with hepatitis D.
(Choice E) Infectious mononucleosis can lead to hepatitis in
adolescents and young adults. Signs and symptoms typically include sore
throat, fever, lymphadenopathy, rash, and splenomegaly.

Educational Objective:
Epidemiological differences (e.g., incubation period, route of
transmission, risk factors) offer some of the best means of identifying
the virus responsible for a patients hepatitis. Confirmation can be
established with serological testing.

__________________________________________________________ 69
USMLE WORLD STEP 2 CK (INTERNAL MEDICINE) *Block
2*
Q NO 61: A 32-year-old female is brought to the clinic by her husband
because he believes she is a malingerer and is just being difficult.
Sometimes, she appears confused and disoriented. Over the past year
she has complained of visual loss, eye pain and inability to do any
household chores. Two months ago, she claimed to have lost control of
her bladder. Interestingly, she is her normal self when it is time to go
for summer trips. The wife insists that she does not understand what is
happening to her, and adds that she occasionally loses the ability to
move her right hand. The physical examination is basically normal. The
patient appears, alert, oriented, and is in no distress. Which of the
following is the most appropriate next step in management?
A. MRI of the brain
B. Lumbar puncture
C. Tonometry
D. Serum immunoglobulins
E. Nerve conduction studies
F. Refer to psychiatry

Explanation:
Suspect multiple sclerosis (MS) in a female who has recurrent attacks
of focal neurologic dysfunction that occur at non- predictable time
intervals. The age of onset is usually between the third and the fourth
decade. The pathology involves demyelination of focal regions in the
white matter of the brain, with a propensity to involve the
periventricular and subpial white matter of the cerebrum, the optic
nerves, brain stem and spinal cord. The MRI is the diagnostic test of
choice for identi1iing demyelinating lesions. Visual, auditory and
somatosensory evoked response tests are of value in identifying silent
lesions.
(Choice B) Abnormalities in the cerebrospinal fluid may be seen with
MS. Such abnormal CSF findings include oligoclonal bands, pleocytosis,
elevated IgG or myelin basic protein and mild lymphocytosis. However,
these findings are not very consistent, and are not diagnostic.
(Choice C) Tonometry is usually done to measure the pressure in the
ocular chambers to assess the presence of glaucoma.
(Choice D) Changes in serum immunoglobulins occur in MS, but are not
specific. Analysis of T lymphocyte subpopulations may demonstrate
reduced numbers of cells with the suppressor phenotype during or
preceding attacks.
(Choice E) Nerve conduction studies do not play a role in the diagnosis
of MS, but these can be used to identify the location of the current
neurologic deficit in the extremities, as well as clinically silent
lesions. Urodynamic studies often aid in the evaluation and management
of bladder symptoms.
(Choice F) Despite the husbands belief that the patient is
malingering, the patients history is suggestive of a neurologic
disorder (MS) which warrants further evaluation with neuroimaging
studies. A psychiatry referral is not warranted at this time.

Educational Objective:
Multiple sclerosis is best diagnosed with MRI. The MRI shows white
matter disease, which is very characteristic for multiple sclerosis.

70 __________________________________________________________
USMLE WORLD STEP 2 CK (INTERNAL MEDICINE) *Block
2*
Q NO 62: A pharmaceutical company researcher is working on drugs x, y &
z proposed for the treatment of diabetes mellitus. He selects a group of 300
diabetic patients from a county hospital in Jackson. TN. He randomly divides
them into 3 groups of 100 patients each (a, b & c). Group a receives drug x,
group b receives drug y, and group c receive drug z. He follows these
groups prospectivelyfor6 months and tabulates the following results in
below graph. Based on these results, which of the following statements is
correct?

A. This study is invalid because the sample selection is improper


B. Drug x is the most potent amongst the three drugs
C. There is no statistically significant difference between the effects
of drug x & z
D. Drug z has the widest range of effects (i.e. it is effective in some
patients & not effective in others)
E. Drug y will result in a significant increase in mortality in
diabetic patients if used on a regular basis

Explanation:
Confidence interval (Cl) of the mean is calculated by the following
formula:
Cl (mean)= mean standard score (z) standard error of mean (SE).
Standard error of mean (SE) is calculated by dividing the standard
deviation (SD) by the square root of the sample size (N). Hence, as the
SD increases, the SE and Cl also increase. As N increases, SE
decreases.
In the abovementioned study, the Cl for group a is 0.8 (85.4 minus
84.6= 0.8). Similarly, the Cl for group b is 0.94, and that of group c
is 1.32. (Note that the Cl is given in the table and you can save time
from calculating the Cl for all 3 groups if you look into all the data
carefully. You can calculate the Cl for all 3 groups using SD, N, and
mean values, but it will be a waste of time during the exam!!!)
A larger confidence interval means that there is a wider range of
possible effects. A smaller confidence interval means that there is a
narrower range of possible effects. Since drug z has the widest
confidence interval, it will have the widest range of possible effects.
(Choice A) The researcher has selected all the patients with diabetes
and has divided them randomly, making this a valid study.
(Choice C) If the confidence intervals of different groups overlap,
then the groups are considered to have no statistically significant
difference. Since confidence intervals of groups a and c are non-

__________________________________________________________ 71
USMLE WORLD STEP 2 CK (INTERNAL MEDICINE) *Block
2*
overlapping, there is a statistically significant difference between
these groups.
(Choices B and E)This study does not give any information about the
drugs potency or the reduction in mortality. Drug potencies are
determined by log dose response curves. Comments about mortality cannot
be made, as mortality rates are not mentioned.

Educational Objective:
Confidence interval (Cl) of the mean is calculated by the following
formula.
Cl (mean) = mean standard score (z) standard error of mean (SE).

72 __________________________________________________________
USMLE WORLD STEP 2 CK (INTERNAL MEDICINE) *Block
2*
Q NO 63: A 54-year-old African American male presents to the FR with
excruciating right flank pain radiating to his testicle for the past 2
hours. He describes this pain as a 10/10. He also complains of
vomiting. His medical history is significant for gout, coronary artery
disease, HIV, Hepatitis C, chronic renal insufficiency from HIV, and
G6PD deficiency. Patient has not been compliant with his medications
except vitamin C. On physical examination, he is extremely restless and
in obvious pain. Abdominal examination is normal except for mild
guarding; genitalia are normal. Plain x-ray abdomen shows bright
radiopaque renal calculi. The most likely cause for the development of
this condition is?

A. Gout
B. HIV
C. Hepatitis C
D. G6PD deficiency
E. Vitamin C

Explanation:
High doses of vitamin C (>1 gm) can precipitate renal calculi by
increasing urinary oxalate excretion. It predisposes to formation of
calcium oxalate stones. Even though the data is tentative, patients
with a predisposition to form oxalate stones or those on hemodialysis
should avoid excessive use of vitamin C.
Gout can cause uric acid stones but they are radiolucent as opposed to
the radiopaque stones in this case.
HIV does not predispose to formation of renal calculi although
indinavir (anti-retroviral drug) can cause nephrolithiasis.
G6PD deficiency and Hepatitis C have no relation to renal calculi.
High doses of Vitamin C can induce hemolysis in patients with G6PD
deficiency.

Educational Objective:
Excessive use of vitamin C in patients with renal insuffiency can cause
oxalate stones.

__________________________________________________________ 73
USMLE WORLD STEP 2 CK (INTERNAL MEDICINE) *Block
2*
Q NO 64: A 70-year-old Caucasian male presents to your office for
evaluation of skin lesions on his forehead. On physical exam you
find that these papules have a sand paper texture by palpation.
The lesions are illustrated in the slide below. Which of the following
is the most likely diagnosis in this patient?

A. Psoriasis
B. Seborrheic keratosis
C. Actinic keratosis
D. Atopic dermatitis
E. Pityriasis rosea

Explanation:
The lesions on the image above represent actinic keratoses (AK). AKs
are classically described as erythematous papules with a central scale
due to hyperkeratosis. A sandpaper-like texture on palpation of the
affected areas is typical for this condition. The lesions are small and
flat at first, but may enlarge and become elevated. Usually their size
does not exceed 10mm in diameter. Hyperkeratosis in such lesions may
become prominent and turn into cutaneous horns.
Actinic keratoses develop in genetically predisposed individuals 40-60
years of age under the influence of excessive sun exposure. The most
commonly affected areas are the face, ears, scalp and the dorsa of the
arms and hands, but any other chronically sun-exposed site (legs, back,
upper chest) can be involved.
On light microscopy, affected areas show acanthosis (thickening of the
epidermis), parakeratosis (retention of nuclei in the stratum corneum),
dyskeratosis (abnormal keratinization), and hyperkeratosis (thickening
of stratum corneum). Keratinocytes display various degrees of atypia.
Mitoses and an inflammatory infiltrate are present. Actinic keratosis
is regarded as either a premalignant condition or a carcinoma in situ,
but less than 1% of AKs will evolve into frank squamous cell carcinoma.
(Choice A) Psoriasis presents with well-circumscribed raised papules
and plaques covered with a thick silvery scale. The lesions are located
on the scalp, trunk and extensor areas of extremities (Elbows and
knees).
(Choice B) Seborrheic keratoses occur in elderly individuals and
presents with stuck-on, deeply pigmented or flesh colored lesions
with velvety or greasy surface.
(Choice D) Atopic dermatitis is a type I (immediate) hypersensitivity
reaction. Acute atopic dermatitis manifests with highly pruritic
papules, vesicles and plaques. Light microscopy demonstrates spongiosis
(edema of the epidermis).
(Choice E) The first symptom of pityriasis rosea is pink or brown scaly
plaque with central clearing and a collarette of scale (herald patch)
on the trunk, neck or extremities. It is followed by development of

74 __________________________________________________________
USMLE WORLD STEP 2 CK (INTERNAL MEDICINE) *Block
2*
maculopapular rash classically in a Christmas tree pattern along the
skin tension lines.

Educational Objective:
Actinic keratoses develop in predisposed individuals on chronically
sun-exposed areas of the skin. The lesions consist of erythematous
papules with a central scale and a sandpaper-like texture. Actinic
keratoses can convert to squamous cell carcinoma in approximately 1% of
cases.

__________________________________________________________ 75
USMLE WORLD STEP 2 CK (INTERNAL MEDICINE) *Block
2*
Q NO 65: A 42-year-old male presents with a 2-year history of heartburn.
The heartburn occurs after heavy meals and while supine. Overtime, his
symptoms have been increasing in severity and frequency. His symptoms
used to be alleviated with the use of over-the-counter antacids. But these
have become ineffective in the past two months. He also complains of
epigastric pain and occasional vomiting, both of recent onset, especially in
the morning. He denies dysphagia or odynophagia. He eats junk food and
drinks two cups of coffee daily. He regularly drinks grape brandy and
smokes 1 pack of cigarettes/day. Abdominal examination shows epigastric
tenderness. Abdominal ultrasound is unremarkable. Test of the stool for
occult blood is negative. Which of the following is the most appropriate
next step in management?

A. Treatment with ranitidine


B. Upper GI endoscopy
C. Barium swallow
D. Manometric studies
E. Reassurance

Explanation:
This patients 2-year history of heartburn is yew suggestive of reflux
esophagitis; however, the recent onset of new symptoms also suggests
another problem, such as Barretts esophagus, peptic ulcer disease,
gastritis, or even a tumor. Of the listed studies, endoscopy can
provide the most information in the work-up of any of these diagnoses.
(Choice A) The generally recommended initial treatment for patients
with uncomplicated gastroesophageal reflux disease is a trial of proton
pump inhibitor therapy. H2 receptor antagonists are no longer
recommended as the first line of therapy for grade 2 or worse
esophagitis. Patients should be advised about anti-reflux measures
(e.g., stopping smoking, weight loss, elevating the head of the bed,
small meals, and modifying the eating habits).
(Choice C) Provided that the patient does not have dysphagia, endoscopy
can be performed without a preceding contrast study.
(Choice D) If endoscopy is negative, manometry may be indicated.

Educational Objective:
Recognize when to order upper endoscopy in the management of
gastroesophageal reflux disease. The following are some alarm signals:
1. Nausea/vomiting
2. Weight loss anemia or melena/blood in the stool
3. Long duration of symptoms (>1-2 years), especially in Caucasian
males >45 years old
4. Failure to respond to proton pump inhibitors

76 __________________________________________________________
USMLE WORLD STEP 2 CK (INTERNAL MEDICINE) *Block
2*
Q NO 66: A 38-year-old female presents to the physician because of a
two-week history of worsening headaches and low-grade fever. She denies
any weakness or sensory loss. She has had significant weight loss over
the last six months. Her pulse is 80/min, blood pressure is 115/70 mm
Hg, respirations are 14/min, and temperature is 38.1 C (100.5 F). She
appears cachectic. Her neck is stiff and there is cervical
lymphadenopathy present. Oropharyngeal exam reveals thrush. The
neurological examination is non1ocal, and funduscopy is within normal
limits. Chest is clear to auscultation. Lumbar puncture and CSF
examination show the following results:
Opening pressure 250 cm H20
Glucose 44 mg/dL
Protein 120 mg/dL
RBC none
WBC 50/mm
CSF cultures grow encapsulated yeast. Which of the following is the
most appropriate next step in the management of this patient?

A. Amphotericin plus flucytosine


B. Intravenous itraconazole
C. Oral griseofulvin
D. Oral sulfadiazinepyrimethamine
E. HIV testing and anti retroviral therapy

Explanation:
This patients clinical features and OSE studies highly suggest
subacute cryptococcal meningitis and underlying HIV infection.
Cryptococcus is an encapsulated yeast that commonly causes meningitis
in HIV patients. Tuberculosis is also an important cause of subacute
meningitis in these patients. Initial induction therapy for central
nervous system cryptococcal infection in AIDS patients is IV
amphotericin B plus oral flucytosine.
When there is clinical improvement with induction therapy amphotericin
and flucytosine are discontinued and oral fluconazole is started as
maintenance therapy.
(Choice B) Itraconazole can be used as a maintenance therapy for
Histoplasma capsulatum meningitis. H. capsulatum meningitis is also
initially treated with amphotericin.
(Choice C) Oral griseofulvin is not appropriate to treat cryptococcal
meningitis.
(Choice D) Oral sulfadiazine-pyrimethamine is used to treat
toxoplasmosis, which usually manifests as encephalitis in HIV patients.
In toxoplasma encephalitis, CT scan with contrast shows multiple
hypodense, ring-enhancing lesions.
(Choice E) HIV testing and, if positive, HAART therapy is indicated in
this patient but should not delay antifungal treatment.

Educational Objective:
IV amphotericin plus flucytosine is the antibiotic regimen of choice
for central nervous system cryptococcal infection in AIDS patients.

__________________________________________________________ 77
USMLE WORLD STEP 2 CK (INTERNAL MEDICINE) *Block
2*
Q NO 67: A 44-year-old obese female undergoes an open cholecystectomy
for a complicated acute cholecystitis. On her third post-operative day,
her temperature is 36.7 C (98.2 F), blood pressure is 110/80mm Hg and
pulse is 92/min. Her arterial blood gas shows the following:
Blood pH 7.28
PaO2 62 mmHg
PaCO2 54mmHg
HGO3- 30mEq/L
What is the most likely cause of acidosis in this patient?

A. Alveolar hypoventilation
B. Acute pulmonary embolism
C. Atelectasis
D. Pulmonary edema
E. Pleural effusion

Explanation:
The patient described has a respiratory acidosis evidenced by her
arterial blood gas values that show a decreased pH (c 7.35) and a
primary increase in PaCO2 (>40 mm Hg). High PaCO2 and low PaO2 levels
are suggestive of alveolar hypoventilation, though an elevated PaCO2
alone, in the range of 50 to 80 mmHg, is sufficient to make the
diagnosis. Causes of alveolar hypoventilation and respiratory acidosis
include the following:
Pulmonary/Thoracic diseases: COPD, obstructive sleep apnea, cystic
fibrosis, obesity (Pickwickian), scoliosis, ankylosis
Neuromuscular diseases: myasthenia gravis, Lambert-Eaton,
poliomyelitis, muscular dystrophies, Guillain-Barre, ALS
Drug-induced hypoventilation: anesthetics, narcotics, sedatives
Primary CNS dysfunction: brainstem lesion, infection, stroke
(Choices B, C, D and E) Acute pulmonary embolism, atelectasis,
pulmonary edema and pleural effusion can cause a decrease in PaO2
(hypoxemia), but these conditions typically also cause a decrease in
PaCO2 (respiratory alkalosis) due to concomitant tachypnea.

Educational Objective:
Respiratory acidosis is characterized by a decreased pH and a primary
increase in the PaCO2. Alveolar hypoventilation is an important cause
of respiratory acidosis.

78 __________________________________________________________
USMLE WORLD STEP 2 CK (INTERNAL MEDICINE) *Block
2*
Q NO 68: A 52-year-old African-American male comes for a regular health
check-up. He is a chronic smoker and has been drinking about 2 beers/day
for the past 10 years. He also drinks about? cups of coffee/day. He was
diagnosed with diabetes 5yrs ago and takes metformin for it. On
examination, he has a Body Mass Index (BMI) of 29 and BP: 130/80 mm
Hg. His random blood sugar is 190 mg/dl. His elder brother died of
pancreatic cancer at age of 58 and he is worried that he might also get
pancreatic cancer. Which of the following interventions would decrease his
risk of pancreatic cancer the most?

A. Stop the alcohol intake.


B. Stop smoking.
C. Stop the coffee intake.
D. Stop metformin.
E. Achieve better control of blood sugar with insulin.

Explanation:
Pancreatic cancer is a highly malignant cancer making it fifth most
common cause of cancer related mortality. It results in death in 98% of
afflicted individuals (98% fatality rate). Risk factors for pancreatic
cancer are:
1. Male sex.
2. Increasing age (50 years).
3. Black race.
4. Cigarette smoking: It is the most consistent risk factor. Pancreatic
cancer is 2-3 times more likely in heavy smokers than in nonsmokers.
5. Chronic pancreatitis.
6. Long-standing diabetes.
7. Obesity.
8. Familial pancreatitis.
9. Pancreatic cancer in a close relative.
The following are not risk factors for pancreatic cancer:
1. Alcohol consumption.
2. Gall stones
3. Coffee intake.
(Choices A and C) Alcohol intake and coffee intake are not risk factors
for pancreatic cancer.
(Choice D) Metformin is not associated with pancreatic cancer.
(Choice E) Long standing diabetes is a risk factor for pancreatic
cancer. However, this patient has been diagnosed with diabetes 5 years
ago and the benefits of smoking cessation in prevention of pancreatic
cancer outweigh the benefits of sugar control.

Educational Objective:
Cigarette smoking is the most consistent reversible risk factor for
pancreatic cancer.

__________________________________________________________ 79
USMLE WORLD STEP 2 CK (INTERNAL MEDICINE) *Block
2*
Q NO 69: An 83-year-old woman presents with a 1-year history of A.
progressively severe crampy abdominal pain after eating. She has
started avoiding food because of the pain. The pain is often
associated with bloating, nausea, vomiting, and diarrhea. Recently,
her stools have been bulky foamy, and greasy. She has had a 15kg
(33lb) weight loss over the past year. Her other medical problems
include hypertension, diabetes mellitus-type 2,
hypercholesterolemia, peripheral vascular disease, coronary artery
disease and myocardial infarction. Social history is not significant.
Abdomen is soft, nontender and non-distended. Abdominal x-ray
and CT scan are unremarkable. Which of the following is the most
likely diagnosis?
Chronic pancreatitis
B. Atherosclerosis of the mesenteric arteries
C. Crohn s disease
D. Irritable bowel syndrome
E. Celiac disease
F. Symptomatic cholelithiasis
G. Amyloidosis
H. Intestinal lymphoma

Explanation:
Numerous disorders may lead to malabsorption resulting in weight loss
and a change in the character of the stools. Many of these are
associated with dyspeptic symptoms, but only a few present with severe
abdominal pain. Worsening postprandial pain that leads to avoidance of
food is characteristic of chronic occlusion of visceral arteries
(abdominal angina). In this vignette, the patient has evidence of
generalized atherosclerosis which further raises suspicion for
atherosclerosis of the mesenteric arteries (Choice B). Routine imaging
studies are usually not informative. Diagnosis requires angiography or
a Doppler ultrasound.
(Choices A and C) Pain accompanied by malabsorption may be due to
chronic pancreatitis or Crohns disease. However, these diseases
usually produce abnormal CT and x-ray findings.
(Choice D) Irritable bowel syndrome (IBS) does not lead to weight loss.
IBS is a diagnosis of exclusion. In this case, you should not make this
diagnosis - especially with the above presentation.
(Choice F) Cholelithiasis presents with right upper quadrant pain and
fatty food intolerance. It does not cause chronic diarrhea and weight
loss.
(Choice E) Celiac disease is due to gluten sensitivity and it usually
presents between 10-40 years of age. These patients present with
features of malabsorption-like chronic diarrhea, steatorrhea and
flatulence. Abdominal angina is not a feature of this entity.
(Choices G and H) Amyloidosis and intestinal lymphoma are unlikely in
the above patient.
Educational Objective:
The diagnosis of chronic mesenteric ischemia is suspected in patients
with unexplained chronic abdominal pain, weight loss, and food
aversion. Evidence of associated atherosclerotic disease is usually
present. Physical findings are usually nonspecific. Abdominal
examination may reveal a bruit (50% of patients).
80 __________________________________________________________
USMLE WORLD STEP 2 CK (INTERNAL MEDICINE) *Block
2*
Q NO 70: A 28-year-old female with severe Crohns disease is unable to
eat. It is recommended that she be placed on total parenteral nutrition.
A left subclavian vein line is placed without any complications. The
infusion is started at a rate of 40cc/hour and increased to 75cc/hour
over the next 24 hours. She continues on total parenteral nutrition for
two weeks and returns with complaints of pain and swelling in her left
arm. On examination, the left arm is swollen and pale. The radial and
ulnar pulses are present. What should be the initial step in her
management?

A. Apply heat to the left arm


B. Start heparin
C. Get chest x-ray
D. Remove catheter
E. Start antibiotics

Explanation:
Thrombosis of the subclavian line or superior vena cava can occur after
total parenteral nutrition lines. Thrombosis is insidious and usually
presents after a couple of weeks. Total parenteral nutrition lines
should generally be placed in the right atrium. The irritating and
hyperosmolar fluid can traumatize the veins and lead to thrombosis. The
patient will usually present with a swollen arm (which is pale) and
pulses present. The swelling may extend from the hand to the neck, and
engorged veins in the upper neck and face may be visible. Because the
process is gradual, most patients will have minimal symptoms. The most
important thing about her management is to remove the catheter. The
longer the catheter stays in, the higher the chance of irreversible
injury (Choice D).
(Choice A) Application of heat and elevation of the arm are important,
but can be done after the catheter is removed. Once the line is
removed, the swelling may take a few weeks to resolve. The obstruction
in the subclavian line can be followed by duplex venous ultrasound.
(Choice B) Once the catheter is removed, the swelling will decrease.
Ultrasound should be ordered to document the thrombus. Presence of
thrombus requires anticoagulation for a short duration. Great care must
be taken in not starting intravenous lines in the affected arm in the
future. Long-term anticoagulation is not necessary, but should be
continued if the symptoms do not resolve.
(Choice C) A chest x-ray is not diagnostic of subclavian vein
thrombosis. The diagnosis is made by duplex ultrasound.
(Choice E) Blood cultures are not necessary, as this is not an
infection but a venous obstruction. If, however, the line tip comes
back as infected, antibiotics maybe required if symptoms of fever
persist.

Educational Objective:
Prolonged placement of central lines can lead to subclavian vein
thrombosis and result in arm swelling. Catheters should be removed and
duplex ordered to document the thrombus and for the need of
anticoagulation.

__________________________________________________________ 81
USMLE WORLD STEP 2 CK (INTERNAL MEDICINE) *Block
2*
Q NO 71: A 65-year-oldwoman presents with complaints of pain and A.
swelling over the inner aspect of her right eye for the past two days.
Examination of the eye reveals tenderness, edema, and redness over
the medial canthus. Slight pressure over the area causes expression
of purulent material. Visual acuity is normal. What is the most likely
diagnosis?
Episcleritis
B. Dacryocystitis
C. Hordeolum
D. Chalazion
E. Orbital cellulitis

Explanation:
Dacryocystitis is an infection of the lacrimal sac. It usually occurs
in infants and adults over the age of 40. Acute dacryocystitis is
characterized by the sudden onset of pain and redness in the medial
canthal region. Sometimes, a purulent discharge is noted from the
punctum. A few patients present with fever, prostration, and an
elevated leukocyte count. Staphylococcus aureus and beta-hemolytic
Streptococcus are the usual infecting organisms. It usually responds to
systemic antibiotic therapy.
(Choice A) Episcleritis is an infection of the episcleral tissue
between the conjunctiva and sclera. Patients complain of an acute onset
of mild to moderate discomfort, photophobia, and watery discharge.
Examination reveals diffuse or localized bulbar conjunctival injection.
(Choice C) Hordeolum refers to an abscess located over the upper or
lower eyelid. Itis usually caused by Staphylococcus aureus. It appears
as a localized red, tender swelling over the eyelid.
(Choice D) Chalazion presents as lid discomfort. It is a chronic,
granulomatous inflammation of the meibomian gland. It appears as a
hard, painless lid nodule.
(Choice E) Orbital cellulitis refers to an infection posterior to the
orbital septum. It is unilateral and more common in children. It is
manifested by an abrupt onset of fever, proptosis, restriction of
extraocular movements and swollen, red eyelids.

Educational Objective:
Dacryocystitis presents with inflammatory changes in the medial canthal
region of the eye. Staphylococcus aureus and beta-hemolytic
Streptococcus are the usual infecting organisms.

82 __________________________________________________________
USMLE WORLD STEP 2 CK (INTERNAL MEDICINE) *Block
2*
Q NO 72: A 53-year-old white male presents with progressively worsening
dyspnea over a period of 4 months. He also complains of loss of appetite
and weight. He has a history of chronic duodenal ulcer and complains of
some epigastric discomfort. He takes ranitidine on and oft. He denies
alcohol intake and smoking. He is a plumber and has been doing this job
for more than 30 years. He denies any exposure to pets. His pulse is
86/min, Blood Pressure is 135/78 mmHg, Temperature is 36.9C, and
Respiratory rate is 15/min. Chest examination shows bilateral crackles.
Chest X-ray shows multiple pleural plaques bilaterally; there is also a mass
on the periphery of the right lung. CT guided biopsy of the mass was done.
Which of the following malignancy is most likely seen on biopsy?

A. Metastatic stomach cancer


B. Metastatic colon cancer
C. Metastatic pancreatic cancer
D. Bronchogenic carcinoma
E. Peritoneal mesothelioma
F. Pleural mesothelioma

Explanation:
Pleural involvement is a hallmark of asbestos exposure; pleural plaques
are seen in up to 50% of patients. His occupation is also puts him at
high risk for asbestosis. Both mesothelioma and bronchogenic carcinomas
are associated with the asbestos exposure but bronchogenic carcinoma is
far more common and occurs more frequently after asbestos exposure.
Cigarette smoking acts synergistically with asbestos exposure in
increasing the risk for bronchogenic carcinoma. Both pleural
mesothelioma and peritoneal mesothelioma can occur after asbestos
exposure.
Colon, pancreatic, and stomach cancer are not found to be associated
with asbestos exposure or asbestosis. His chronic duodenal ulcer might
be the cause of his epigastric discomfort. Duodenal ulcer never causes
malignancy.

Educational Objective:
Remember bronchogenic carcinoma is the most common lung cancer
associated with asbestos exposure while malignant mesothelioma is
almost exclusively associated with asbestos exposure but is not the
most common malignancy after asbestos exposure.

__________________________________________________________ 83
USMLE WORLD STEP 2 CK (INTERNAL MEDICINE) *Block
2*
Q NO 73: A 33-year-old female presents to the office for the evaluation of a
one-week history of lightning-like pain on the left side of her face. The pain
is very sharp and feels like a burn. An episode lasts for 10 seconds, occurs
10-20 times a day, and keeps her from sleeping, eating, or working. She
denies any history of trauma, medication use or recent surgery. Vital signs
are within normal limits. What is the most likely diagnosis?

A. Maxillary sinusitis
B. Carotidynia
C. Trigeminal neuralgia
D. Herpes zoster
E. Burning mouth syndrome

Explanation:
Trigeminal neuralgia is characterized by pain in the distribution of
the branches of the fifth nerve. The diagnosis is generally clinical
and based on the characteristic pain, which is severe, intense, burning
or electric shock-like. This symptom occurs in paroxysms that last a
few seconds to minutes each, but occurs many times a day. The trigger
factors may vary, and the cause remains unknown. Carbamazepine is the
drug of choice.
(Choice A) Maxillary sinusitis can also produce facial pain; however,
the pain intensity is much less than trigeminal neuralgia, constant,
and localized to the maxillary sinus. Furthermore, patients usually
have an associated fever and nasal secretions. Sinus x-rays may show
opacification of the maxillary sinus, and treatment involves
antibiotics and/or drainage.
(Choice B) Carotidynia is a neurological condition caused by
inflammation of the carotids and the vagus. The pain is sharp and
localized to the carotid artery distribution in the neck. Usually, the
intensity of pain is much less. The diagnosis is clinical, although an
MRI is sometimes required.
(Choice D) Herpes zoster can also produce excruciating pain: however,
the virus usually occurs along a dermatomal distribution and presents
with vesicles or a macular rash.
(Choice E) Burning mouth syndrome is a rare cause of facial pain. Itis
caused by a virus and the individual has reddened mucosa and has
significant pain. The condition is aggravated by dryness. It is treated
by supportive care.

Educational Objective:
Paroxysmal, lightning-like pain on the face is usually due to
trigeminal neuralgia. Carbamazepine is the drug of choice.

84 __________________________________________________________
USMLE WORLD STEP 2 CK (INTERNAL MEDICINE) *Block
2*
Q NO 74: A 53-year-old man comes to the office for his routine follow-up
visit. He is a known hypertensive for the past 15 years, and is on lisinopril
and hydrochlorothiazide. His blood pressure is 150/90 mm Hg, pulse is
82/min, respirations are 14/min, and temperature is 98.4 F (36.88 C).
Visual acuity is normal. Funduscopy reveals narrow, tortuous retinal
arterioles, copper wiring, and AV nicking. The rest of the examination is
normal. Which of the following is the most likely diagnosis?

A. Normal retina
B. Grade I Hypertensive Retinopathy
C. Grade II Hypertensive Retinopathy
D. Grade III Hypertensive Retinopathy
E. Grade IV Hypertensive Retinopathy

Explanation:
Systemic hypertension affects the retinal circulation as well as the
choroidal circulation. The sequence of events begins with focal spasm
of arterioles, followed by progressive sclerosis and narrowing. If
hypertension is left uncontrolled, this eventually leads to flame-
shaped hemorrhages from ruptured vessels, formed exudates and
papilledema.
Keith-Wagener classification:
Grade I Slight AV nicking
Grade II Copper wiring, AV depression with humping ends
Grade III Silver wiring, flame shaped hemorrhages, exudates
Grade IV Flame shaped hemorrhages, exudates and papilledema

Educational Objective:
Know the various stages of hypertensive retinopathy.

__________________________________________________________ 85
USMLE WORLD STEP 2 CK (INTERNAL MEDICINE) *Block
2*
Q NO 75: A 29-year-old male with a 6-year history of HIV infection A.
presents with chronic, severe diarrhea associated with malaise,
nausea, anorexia and abdominal cramps. His last CD4 count was 80
cells/mm. A modified acid-fast stain of a stool specimen shows 4-
6mm oocysts. Which of the following is the most likely
microorganism responsible for this condition?
Mycobacterium avium complex
B. Cryptosporidium parvum
C. Isospora belli
D. Pneumocystis carinii
E. Microsporidia

Explanation:
A modified acid-fast stain showing oocysts in the stool is very
suggestive of an infection with Cryptosporidium parvum. This organism
can cause severe diarrheal disease in both immunocompetent and
immunocompromised individuals. HIV-infected patients with a more
preserved CD4 count tend to have a self-limiting illness, whereas AIDS
patients with CD4 counts <, 80 cells/mm3 tend to have a more persistent
clinical course.
(Choice C) Although the diagnosis of Isospora belli is also suggested
by the presence of acid-fast oocysts, infections with Isospora belli
are not as common as Cryptosporidium in the United States.
(Choice A) Mycobacterium avium complex is associated with lung
infections in immunocompetent patients with chronic lung disease;
however, it causes disseminated disease with bowel infiltration and
malabsorption in patients with severe immune compromise. Furthermore,
it is not characterized by oocysts in the stool.
(Choice D) Pneumocystis carinii is associated with pulmonary disease,
not diarrhea.
(Choice E) The Microsporidia organisms, along with Enterocytozoon
bieneusi and Encephalitozoon intestinalis, are rare causes of diarrhea
in immunocompetent hosts; however, these are associated with severe
malabsorption and persistent diarrhea in HIV-infected patients.
Furthermore, microsporidiosis is characterized by the presence of
spores and not oocysts in the stool.

Educational Objective:
Cryptosporidium parvum is a major cause of chronic diarrhea in HIV-
infected patients with CD4 counts less than, 80 cells/mm3.

86 __________________________________________________________
USMLE WORLD STEP 2 CK (INTERNAL MEDICINE) *Block
2*
Q NO 76: A 26-year-old man presents to the physicians office with a two-
day history of multiple symptoms, including rash on his trunk, headache,
fatigue, malaise, myalgias, and high-grade fever. The rash is not
associated with pain, itching, or burning. It has expanded over the last
two days. He went on a camping trip in Vermont two weeks ago, and
recalls a tick bite at that time. There is a single lesion on his trunk, which
is erythematous with central clearing. The rest of the examination is
unremarkable. What is the most appropriate next step in the management
of this patient?

A. Perform ELISA for confirmation of Lyme disease


B. Perform western blot for confirmation of Lyme disease
C. Give him oral doxycycline
D. Give him oral amoxicillin
E. Give him intravenous ceftriaxone

Explanation:
This patient has many classic features of early-localized Lyme disease,
which include erythema migrans (EM) and nonspecific complaints such as
malaise, headache, muscle pain joint pain, and lymphadenopathy. In
affected patients. EM usually occurs within one month of the tick bite.
The EM rash appears on the arms and moist areas of the body, such as
the axillae, groin, and/or trunk. The EM rash is erythematous and may
spread with a zone of central clearing (giving the classic bulls eye
appearance) or may be uniformly red. It usually does not produce
symptoms such as pain, itching, or burning. EM is pathognomonic for
Lyme disease: therefore, when a patient has typical EM, the next step
is pharmacological treatment with oral doxycycline.
(Choices A and B) The diagnosis of early-localized Lyme disease is
purely clinical; therefore, there is no need to perform serologic tests
in a patient with classic EM, especially if there is a clear history of
tick bite. Serologic testing is recommended for patients with an
intermediate pretest probability of Lyme disease and should include
ELISA, followed by western blot if results are positive or equivocal.
Serologic tests are usually negative in early-localized Lyme disease.
(Choice D) Oral doxycycline is generally preferred over amoxicillin
because it is also effective against possible coexistent ehrlichiosis,
which is spread by the same vector Lyme disease. In pregnant women,
however, amoxicillin is the treatment of choice.
(Choice E) Intravenous therapy with ceftriaxone is generally indicated
for early-disseminated and late Lyme disease.

Educational Objective:
Erythema chronicum migrans in a patient with a tick bite history
warrants immediate treatment with doxycycline. Serologic testing is not
necessary to confirm a Lyme disease diagnosis if the patient presents
with classic FM.

__________________________________________________________ 87
USMLE WORLD STEP 2 CK (INTERNAL MEDICINE) *Block
2*
Q NO 77: A 42-year-old male is brought to the emergency department
immediately after having a prolonged seizure episode. His family
describes a past medical history of grand mal seizures. He has been on
phenytoin for the past 10 years, but stopped taking the drug six months
ago because he had not had any seizures in the last nine years. He is
otherwise healthy and had been doing well until this seizure episode.
He does not use tobacco, alcohol or drugs. He is afebrile. His blood
pressure is 136/88 mm Hg, respirations are 18/min and pulse is 96/min.
He appears confused and lethargic. Chest auscultation is unremarkable,
and his abdomen is soft and nontender. A limited neurologic examination
is non-focal. His laboratory report shows:
Serum sodium 140 mEq/L
Serum potassium 40 mEq/L
Chloride 103 mEq/L
Bicarbonate 17 mEq/L
Blood urea nitrogen (BUN) 20 mg/dL
Serum creatinine 0.8 mg/dL
Blood glucose 98 mg/dL
Chest x-ray and urinalysis are within normal limits, and a CT scan of
the head is unremarkable. Which of the following is the most
appropriate next step in the management of this patients metabolic
acidosis?

A. Give intravenous bicarbonate


B. Check serum ketones
C. Check serum lipase
D. Observe and repeat the labs after 2 hours
E. Start dopamine

Explanation:
This patient has an anion gap metabolic acidosis following a grand mal
(tonic-clonic) seizure. A post-seizure anion gap metabolic acidosis is
classically the result of lactic acidosis. Seizures result in the
accelerated production of lactic acid in the muscle and reduced hepatic
lactate uptake. This post-ictal lactic acidosis is transient and
typically resolves within 60 to 90 minutes. In this patient, the most
appropriate treatment is observation and repeating the chemistry panel
after 2 hours to see if the acidosis has resolved on its own. If it has
not resolved. It is best to look for other potential causes of
metabolic acidosis.
(Choice A) The use of bicarbonate in the treatment of lactic acidosis
or ketoacidosis is very controversial. It is recommended only in severe
acute acidosis (pH < 7.2). Full correction with bicarbonate should not
be sought only a sufficient amount of bicarbonate should be given to
correct the pH to 7.2. In lactic acidosis, bicarbonate treatment may
paradoxically depress cardiac performance and worsen the acidosis by
enhancing lactate production.
(Choice B) The common causes of ketosis include diabetes, alcoholism
and starvation. The patients history does not indicate a risk for
ketosis.
(Choice C) Serum lipase levels are generally used to assess for
pancreatic disease, but they may also be elevated in diabetic
ketoacidosis and other conditions making the test poorly specific for
any one disease process.
(Choice E) The patients blood pressure is within normal limits.
Dopamine is a positive inotrope and a vasoconstrictor used in patients

88 __________________________________________________________
USMLE WORLD STEP 2 CK (INTERNAL MEDICINE) *Block
2*
with hypotension who do not respond to less aggressive measures such as
intravenous fluids. Hypotension with poor end organ perfusion may cause
lactic acidosis, and dopamine may be used in such a scenario.

Educational Objective:
Postictal lactic acidosis commonly occurs following a tonic-clonic
seizure. It is a transient anion gap metabolic acidosis that resolves
without treatment within 60 to 90 minutes following resolution of
seizure activity.

__________________________________________________________ 89
USMLE WORLD STEP 2 CK (INTERNAL MEDICINE) *Block
2*
Q NO 78: A 48-year-old woman comes to the office because her regular
screening mammogram revealed irregular densities. Radiographic-guided
needle biopsy shows evidence of invasive lobular carcinoma. The tumor
is hormone receptor positive (HR +). Further investigation reveals
involvement of the axillary lymph nodes; the other breast seems to be
spared. No distant metastasis is detected. She undergoes modified
radical mastectomy, along with radiotherapy. She is then prescribed
adjuvant chemotherapy and tamoxifen for5years. This patients use of
tamoxifen increases her risk for developing which of the following
cancers

A. Ovarian cancer
B. Endometrial cancer
C. Lung cancer
D. Pancreatic cancer
E. Cervical cancer

Explanation:
Current research studies have shown that the use of the anti-estrogen,
tamoxifen, as an adjuvant for early stage breast cancer reduces the
risk of recurrence of the original cancer, as well as the risk of
developing new cancer in the other breast. Nevertheless there is some
risk associated with tamoxifen use. Tamoxifen increases the risk of two
types of potentially life-threatening uterine cancer (,) endometrial
cancer, which arises in the lining of the uterus, and (2) uterine
sarcoma, which arises in the muscular wall of the uterus. The Breast
Cancer Prevention Trial (BCPT) showed that women who took tamoxifen had
more than twice the chance of developing endometrial cancer, compared
to women who took placebo. Interestingly, most of the endometrial
cancers have been found in the early stages, and treatment has usually
been effective: however, the disease may be life-threatening in some
patients. Furthermore, women who are taking tamoxifen but have had a
hysterectomy are not at increased risk for these cancers.

Educational Objective:
Tamoxifen has a mixed agonist and antagonist activity on estrogen
receptors. Estrogenic effects of tamoxifen increase the risk of
endometrial cancer and venous thrombosis.

90 __________________________________________________________
USMLE WORLD STEP 2 CK (INTERNAL MEDICINE) *Block
2*
Q NO 79: A 65-year-old man presents with a ,-year history of impotence
and decreased libido. He has a ,5-year history of type 2 diabetes
mellitus, which is controlled with diet. He denies any cardiac
problems, visual changes or symptoms of neuropathy. The genitourinary
examination reveals decreased testicular size and absent gynecomastia.
The rest of the physical examination is unremarkable. The laboratory
report shows:
Hemoglobin A1c 5%
Testosterone 2.0 ng/dL (Normal 3-,0 ng/dL)
LH 3U/L
FSH 4U/L
What is the best next step in the management of this patient?

A. Insulin therapy
B. Angiography of the deep arteries of the penis
C. Doppler ultrasonography of penile blood flow
D. Measure the serum prolactin level
E. Measure the serum estradiol level

Explanation:
Erectile impotence in diabetic patients may be due to multiple reasons,
including autonomic neuropathy, medications, functional hypogonadism,
and problems with penile circulation. It is thus essential to have a
broad differential diagnosis to appropriately include all plausible
causes of the patients symptoms. Functional hypogonadism is
characterized by low testosterone and low gonadotrophin (LH and FSH)
levels in the presence of a significant systemic illness (e.g.,
uncontrolled diabetes); the underlying pathology is defective
gonadotrophin-releasing hormone (GnRH) secretion. In contrast, primary
(testicular) hypogonadism is characterized by elevated serum
gonadotrophin levels.
In this case the patients diabetes is well-controlled, as evidenced by
his normal Hb A1c level. At this point, the most likely diagnosis is
secondary (central) hypogonadism, which is characterized by
hypogonadism, low testosterone levels and inappropriately normal
gonadotrophin levels. Measurement of serum prolactin levels is the most
important biochemical test to perform in patients with suspected
central hypogonadism. Regardless of the cause, high serum prolactin
levels inhibit the release of GnRH, thereby resulting in hypogonadism.
Prolactin-secreting pituitary tumor is one of the most important causes
of elevated prolactin levels.
(Choice E) In primary hypogonadism, supranormal serum FSH and LH
concentrations stimulate testicular aromatase activity, thereby leading
to increased estradiol production.
(Choice A) The patients normal Hb A1c level indicates that his
diabetes is well controlled with his current regimen; therefore,
insulin treatment is not warranted.
(Choices C and B) Duplex Doppler ultrasonography or angiography of the
penile deep arteries maybe indicated when a vascular etiology of
erectile impotence is being suspected. Since the patients features are
more suggestive of central hypogonadism, these tests should not be
utilized at this time.
Educational Objective:
Central or secondary hypogonadism in males is characterized by low
levels of testosterone with low or inappropriately normal serum
gonadotropin levels.

__________________________________________________________ 91
USMLE WORLD STEP 2 CK (INTERNAL MEDICINE) *Block
2*
Q NO 80: A 40-year-old Caucasian man comes to the emergency A.
department because of fever, dry cough, and shortness of breath.
Symptoms started 24 hours ago. He denies hemoptysis. He was
recently discharged from the hospital after a second cycle of
chemotherapy for acute myeloid leukemia. He does not use
tobacco, alcohol, or drugs. His temperature is 38.9 C (102.0 F),
blood pressure is 120/70 mmHg, pulse is 112/min and
respirations are 28/min. The patients pulse oximetry showed 86%
at room air. Examination shows diffuse crackles all over the lung
fields. His chest x-ray shows diffuse interstitial infiltrates. Which
of the following is the most likely cause of his condition?
Coccidioidomycosis
B. Histoplasmosis
C. Tuberculosis
D. HIV infection
E. Pneumocystis carinii
F. Hodgkins lymphoma
G. Berylliosis
H. Silicosis
I. Metastatic testicular cancer
J. Bronchogenic carcinoma
K. Sarcoidosis
L. Rheumatoid nodule
M. Wegeners granulomatosis
N. Aspergillosis
O. Candida pneumonia

Explanation:
The patient in this vignette is immunocompromised due to his
chemotherapy, which put him at high risk for numerous opportunistic
infections. The clinical presentation suggests atypical pneumonia,
which along with characteristic chest x ray findings is suggestive of
pneumocystis carinii (P. Carinii) pneumonia.
Pneumocystis carinii is an opportunistic pathogen, and an important
cause of pneumonia in immunocompromised hosts. Its natural habitat is
the lung, and the incubation period ranges from 4-8 weeks. It is
airborne transmitted.
P. Carinii pneumonia is characterized by dyspnea, fever, and a
nonproductive cough. Physical examination would show tachypnea,
tachycardia, and cyanosis with minimal chest findings. Bilateral
diffuse interstitial infiltrates beginning in the perihilar region is a
characteristic finding on chest x-ray in P. Carinii pneumonia.
P. Carinii may disseminate outside the lung, and the most common sites
of extrapulmonary involvement are the lymph nodes, spleen, liver, and
bone marrow.
(Choice N) Aspergillosis can also be seen in immunocompromised
patients; however, the chest x-ray typically shows a solid mass
surrounded by a radiolucent crescent (crescent sign, Monods sign).
(Choice 0) Candida is an extremely rare organism to cause pneumonia in
any patient. It is often colonized in bronchial epithelium and
sometimes the sputum and culture may be positive; however, it is not
the cause of pneumonia. Always look for something else. Think about

92 __________________________________________________________
USMLE WORLD STEP 2 CK (INTERNAL MEDICINE) *Block
2*
candida if the question is talking about esophagitis, endocarditis,
endophthalmitis, hepatosplenic disease, and meningitis.

Educational Objective:
P. Carinii is an opportunistic pathogen, and an important cause of
pneumonia in immunocompromised hosts. Bilateral diffuse interstitial
infiltrates beginning in the perihilar region is a characteristic
finding on chest x-ray.

__________________________________________________________ 93
USMLE WORLD STEP 2 CK (INTERNAL MEDICINE) *Block
2*
Q NO 81: A 34-year-old white woman comes to the Emergency Room
complaining of difficult breathing that started suddenly several hours
ago. She has never had such symptoms before. Her past medical hi
stow is significant for rheumatoid arthritis. Her current medications
include naproxen and an oral contraceptive. She smokes 1 packs a
day, but does not consume alcohol. Her blood pressure is 110/70
mmHg and heart rate is 105/min. Lungs are clear on auscultation.
EKG and chest x-ray are normal. Ventilation/perfusion scan was
performed, but the results were inconclusive. What is the best next
step in the management of this patient?
A. Pulmonary angiography
B. Pulmonary function tests
C. Venous ultrasonography
D. Contrast phlebography
E. Transesophageal echocardiography

Explanation:
This patient is undergoing a diagnostic work-up for pulmonary
thromboembolism. The usual source of emboli causing this condition is deep
venous thrombosis (DVI) of the lower extremities. Oral contraceptive use is
an important risk factor for DVP, especially when combined with smoking.
Pulmonary thromboembolism is difficult to diagnose and usually requires the
combination of several diagnostic modalities. You should be familiar with
the overall scheme of diagnostic work-up in these patients. The first test
that is usually employed is ventilation/perfusion scanning. If the classic
pattern of mismatched perfusion defect is observed, you should proceed with
treatment. If the scan is normal, you can actually rule out significant
pulmonary thromboembolism. But, in a substantial number of patients, the
ventilation/perfusion scanning results are inconclusive. The next best step
in this case is venous ultrasonography to reveal DVT or CT angiogram of the
chest. Diagnosing DVT makes the probability of pulmonary thromboembolism
very high, and you should proceed with treatment.
Although considered the gold standard, pulmonary angiography (Choice A),
an invasive procedure is employed if venous ultrasonography or CT angiogram
is negative. Chest CT angiogram is also frequently used in indeterminate
cases of pulmonary embolism, but the problem with CT angiogram is that it
can diagnose only large emboli. Small emboli can be easily missed.
Contrast phlebography (Choice D) is an invasive procedure that was largely
replaced by venous ultrasonography.
Transesophageal echocardiography (Choice E) is very useful is diagnosing
aortic dissection, but has no role in diagnosing pulmonary thromboembolism.
Pulmonary function tests (Choice B) are useful for diagnosing obstructive
and restrictive disease. They have no role in this patient at this time.

Educational Objective:
If ventilation/perfusion scan is inconclusive in patients undergoing
diagnostic work-up for pulmonary thromboembolism, order venous
ultrasonography to reveal DVT or CT angiogram. Pulmonary angiography is an
invasive procedure and should be employed only if venous ultrasonography or
CT angiogram is negative.

94 __________________________________________________________
USMLE WORLD STEP 2 CK (INTERNAL MEDICINE) *Block
2*
Q NO 82: A 42-year-old Caucasian female presents to clinic complaining
of severe fatigue and dark yellow urine. Further inquiry reveals that
she is also experiencing anorexia, nausea, and malaise. She is
uncertain as to when her symptoms began and believes they came on
gradually. She has no chronic illnesses, takes no medications, and has
no known drug allergies. She admits to having unprotected intercourse
with six different partners within the past year. Her immunization hi
story does not include vaccination against hepatitis B virus.
Laboratory testing reveals the following:
Liver studies
Total bilirubin 4.5 mg/dL
Alkaline phosphatase 142 U/L
Aspartate aminotransferase (ASTI SGOT) 184 U/L
Alanine aminotransferase (ALT, SGPT) 345 U/L
Which of the following is the best means of screening for hepatitis B
infection?

A. HBsAg and IgM anti-HBc


B. HBsAg and anti-HBs
C. HBcAg
D. HBeAg
E. Anti-HBe

Explanation:
Expected laboratory findings in a patient with acute hepatitis include
significant elevations in ALT and AST (with ALT> AST), followed by
rises in bilirubin and alkaline phosphatase. Because this patients
liver function tests are suggestive of acute hepatitis and she reports
history of multiple sex partners, she is at high risk for infection
with hepatitis B virus and should be screened accordingly.
The serological markers of hepatitis B develop over a specific time
course, as outlined below.
HBsAg: The first virological marker detected in the serum after
inoculation, it precedes both the elevation of serum aminotransferases
and the onset of clinical symptoms. It remains detectable during the
entire symptomatic phase of acute hepatitis B and suggests infectivity.
Anti-HBs: Appearing in the serum after either successful HBV
vaccination or the clearance of HBsAg, this marker remains detectable
for life. It serves as an indicator of non infectivity and immunity.
However, there is a time lag between the disappearance of HBsAg and the
appearance of anti-HBs in the serum, which is termed the window period
HBcAg: This marker is not detectable in serum as it is normally
sequestrated within the HBsAg coat.
Anti-HBc: Appearing in the serum shortly after the appearance of HBsAg,
this marker remains detectable long after the patient recovers. The ,gM
fraction signals the acute phase of disease, whereas the lgG fraction
signals recovery from the disease. Because IgM anti-HBc is present in
the window period, itis an important tool for diagnosis when HBsAg
has been cleared and anti-HBs is not yet detectable. Thus ,gM anti-HBc
is the most specific marker for diagnosis of acute hepatitis B.
HBeAg: This antigen is detectable shortly after the appearance of HBsAg
and indicates active viral replication and infectivity. It is
associated with the presence of HBV DNA. HBeAg tends to disappear
shorlly after aminotransferase levels peak and before HBsAg is
eliminated, and is followed by the appearance of anti-HBe. Should it

__________________________________________________________ 95
USMLE WORLD STEP 2 CK (INTERNAL MEDICINE) *Block
2*
persist for more than three months, there is an increased likelihood of
chronic hepatitis B.
Anti-HBe: This marker suggests the cessation of active viral
replication and low infectivity.
Therefore, testing for both HBsAg and anti-HBc offers the best
screening for acute hepatitis B infection, as it wont miss the window
period when HBsAg has disappeared but anti-HBs has not yet appeared in
the serum.
(Choice B) Testing for HBsAg and anti-HBs will not diagnose hepatitis B
infection in those patients who are in the window period.
(Choice C) HBcAg is not detectable in serum and is therefore of no help
in establishing the diagnosis.
(Choice D) HBeAg is a good indicator of infectivity but is a poor
screening tool as its levels typically fall early in the course of the
disease.
(Choice E) Anti-HBe is a good indicator that viral replication has
ceased. However, its sensitivity is too poor to diagnose an acute
infection.

Educational Objective:
Testing for both HBsAg and anti-HBc offers the best screening for acute
hepatitis B infection.

96 __________________________________________________________
USMLE WORLD STEP 2 CK (INTERNAL MEDICINE) *Block
2*
Q NO 83: A 26-year-old man comes to the office with a one-week history of
right-sided ear pain. The pain often wakes him up at night, and increases in
severity when he chews food. He cannot recall any episodes of pharyngitis.
He denies having any ear discharge, sinus tenderness, or skin rash. He
exercises by swimming frequently at a local club. He is sexually active, and
uses condoms quite regularly. He lives with his brother, who often
comments on his habit of grinding his teeth at night. On examination, his
ears seem normal with mild wax. Pain is not elicited by pulling the pinna.
There are no hearing defects appreciated. Mobility of the tympanic
membrane is normal. Weber and Rinne test results are within normal limits.
What is the most likely diagnosis?

A. Ramsay Hunt syndrome


B. Glossopharyngeal neuralgia
C. Otitis media in early stages
D. Tempomandibular joint dysfunction
E. Otitis externa
F. Cerumen Impaction

Explanation:
The most likely diagnosis is referred otalgia due to temporomandibular
joint (TMJ) dysfunction. Always think of referred otalgia when a
patient presents with ear pain, but has no significant or prior history
of ear problems. TMJ dysfunction is a common cause of referred otalgia,
and pain aggravated by chewing, along with psychogenic grinding
(bruxism), further supports the diagnosis.
(Choice A) Ramsay Hunt syndrome is a facial nerve palsy that is caused
by herpes zoster. Patients present with lesions of the external ear
with or without tympanic membrane involvement vertigo tinnitus, and
deafness due to geniculate ganglion involvement.
(Choice B) Glossopharyngeal neuralgia is characterized by a sharp,
lancinating pain associated with swallowing, chewing, talking and
yawning. This condition has also been associated with multiple
sclerosis.
(Choice C) Otitis media is associated with reduced mobility of the
tympanic membrane and involvement of the middle ear cavity.
(Choice E) Although the patients history of frequent swimming
increases his chances of having otitis externa, the physical
examination failed to find any evidence that can support this diagnosis
(e.g., pain on pulling the pinna or inflamed ear canals).
(Choice F) Cerumen impaction is very unlikely in this setting,
especially since the tympanic membrane was normally visible and without
signs of cerumen impaction on physical examination.

Educational Objective:
Suspect TMJ syndrome in a patient with unilateral and chronic pain of
the muscles of mastication. The pain is dull and worsened by chewing.
Recognize the characteristic radiation to the ear, jaw, and posterior
cervical region.

__________________________________________________________ 97
USMLE WORLD STEP 2 CK (INTERNAL MEDICINE) *Block
2*
Q NO 84: A 25-year-old man presents to the emergency department after a
sudden onset of nausea three hours ago, followed by six or seven episodes
of vomiting. He denies any diarrhea or fever. Four hours ago, he ate fried
rice left over from dinner at a Chinese restaurant. His pulse is 90/min, blood
pressure is 100/80mm Hg, and temperature is 37.1 C (98.8 F). There are
no significant findings on physical examination. Which of the following is the
most likely cause of this patients symptoms?

A. Staphylococcus aureus
B. Bacillus cereus
C. Clostridium perfringens
D. Clostridium difficile
E. Enterotoxigenic E. coil

Explanation:
For the USMLE, suspect Bacillus cereus whenever you read about a
patient who eats rice and subsequently develops nausea and severe
vomiting. Bacillus cereus produces a heat-stable toxin in inadequately
refrigerated cooked rice. Because the illness is due to a preformed
toxin, symptoms of nausea and vomiting appear quickly after consumption
of the contaminated food (between one and six hours after ingestion).
Aside from preformed toxins, chemical irritants also produce abrupt-
onset nausea and severe vomiting.
(Choice A) Staphylococcus aureus toxin is present in foods such as
dairy, salad, meat, and eggs. Symptoms include nausea, vomiting,
diarrhea, and abdominal pain. Because S. aureus food poisoning is also
due to a preformed toxin, symptom-onset is rapid, usually within one to
six hours after ingestion.
(Choice C) Clostridium perfringens is a spore-forming organism. Its
spores germinate in foods such as meats, poultry, or gravy. Ingestion
of such food results in watery diarrhea due to production of toxin in
the gut. Symptom onset is later than with preformed toxins (8-14 hours
after ingestion). Diarrhea occurs with ingestion of a large number of
organisms.
(Choice D) Clostridium difficile causes pseudomembranous colitis
associated with the use of antibiotics.
(Choice E) Enterotoxigenic E. coli is one of the most common causes of
travelers diarrhea, but it is a rare cause of diarrhea in the US.

Educational Objective:
Bacillus cereus causes nausea and vomiting after eating rice. Symptom
onset is within one to six hours.

98 __________________________________________________________
USMLE WORLD STEP 2 CK (INTERNAL MEDICINE) *Block
2*
Q NO 85: A study was done to evaluate a new serological screening test
for HIV infection. 1,000 people were randomly selected from the
population, screened using the new test, and screened again using the
gold standard (i.e., Western blot) to determine his/her true infection
state. The findings are as follows:
True HIV infection state
Test results Yes No Total
Positive 140 60 200
Negative 40 760 800
Total 180 820 1000
What is the sensitivity of this new serological screening test?

A. 140/180
B. 140/200
C. 60/820
D. 60/200
E. 180/1000

Explanation:
Sensitivity and specificity are used to measure a tests validity.
Sensitivity is defined as the proportion of diseased subjects among
those with a positive test result. Specificity is defined as the
proportion of disease-free subjects among those with a negative test
result.
Consider the following 2 x 2 table:

Sensitivity = TP/TP+FN (A/A+C)


Sensitivity determines how well the test identifies the people with the
disease. It is usually the first test used, like a screening test when
there is a public health threat. In other words, the true positives
rate describes the sensitivity of the test.
Specificity =TN/N+FP (D/B+D)
Specificity determines how well the test identifies the healthy people.
In other words, the true negatives rate describes the specificity of
the test.
A perfectly valid diagnostic test should have sensitivity and
specificity equal to, but this is seldom possible. Generally, the more
sensitive the test, the less specific and vice versa.
In the present example, the sensitivity is 140/180.

Educational objective:
Know how to calculate the sensitivity and specificity of a test.

__________________________________________________________ 99
USMLE WORLD STEP 2 CK (INTERNAL MEDICINE) *Block
2*
Q NO 86: A 33-year-old male is admitted in the hospital for elective
cholecystectomy. He has no allergies and denies the use of any
medications. He smokes and uses alcohol liberally. A preoperative
chest x-ray shows a widened mediastinum and an MRI is ordered.
The MRI is shown below. Which of the following is the most likely
cause of his pathology?

A. Atherosclerosis
B. Marfans syndrome
C. Blunt trauma
D. Takayasus disease
E. Mycotic infection

Explanation:
The MRI shows an aortic aneurysm just distal to the left subclavian
artery. The aneurysm is localized to the descending aorta. In a young
male who presents with an aneurysm of the descending aorta, the most
likely cause is a history of prior blunt chest trauma. The dominant
pathophysiology event in blunt aortic injury is due to deceleration
with creation of a shear force between a relatively mobile part of the
thoracic aorta and an adjacent fixed segment. The majority of blunt
injuries to the thoracic aorta are from motor vehicle accidents.
Physical exam in patients with blunt aortic injury are non-contributory
and the injury may be suspected from a widened mediastinum on a chest
x-ray. About 10% of individuals with aortic injury have a normal chest
x-ray. If the injury is suspected, a CT scan is diagnostic. Some
patients maybe discharged on the basis of a normal chest x-ray and
present later with a pseudoaneurysm of the aorta. Surgery is required
to prevent rupture of the aneurysm (Choice C).
(Choice A) Atherosclerosis is the most common cause of aneurysm of the
descending aorta. However, the patients are much older, have a history
of smoking, and peripheral vascular disease. These descending aneurysms
are generally asymptomatic and may be identified on a routine chest x-
ray. Rarely these aneurysms may impinge on the recurrent laryngeal
nerve and produce hoarseness, or compress the esophagus and produce
dysphagia. CT scans or angiograms are diagnostic. The majority of
patients with descending aortic aneurysms also have significant
coronary artery disease.
(Choice B) Martans syndrome is an autosomal dominant disorder with
variable penetrance. It has been found to be associated with an
abnormal synthesis of fibrillin, a major constituent of microfibrils.
The majority of patients with Marfans syndrome will present with

100 __________________________________________________________
USMLE WORLD STEP 2 CK (INTERNAL MEDICINE) *Block
2*
aneurysms of the ascending aorta. Associated findings may be aortic
annular dilatation that will cause aortic regurgitation. Surgery is
required to replace both the aortic valve and entire ascending aorta.
Marfans patients have a higher incidence of aortic dissections than
the average patient.
(Choice D) Takayasus arteritis commonly afflicts young females from
the Orient. The disorder is associated with a prodrome of fever,
malaise, and lethargy. The arterial pathology is focused on the aorta
and its branches. The aortic arch is more frequently involved and leads
to symptoms of global cerebral hypoperfusion or upper extremity
claudication. Aneurysms are not seen in Takayasus disease. Steroids
and balloon angioplasties are the mainstay of treatment.
(Choice E) Mycotic aneurysms result from a localized infection, which
maybe blood borne or associated with a localized infectious process. It
may occur any where, but is most common in the femoral artery followed
by the ascending aorta. The most common pathogen is Staph aureus
followed by Salmonella. Syphilitic aneurysms were common in the past
decade but are rare today. Syphilitic aneurysms are generally confined
to the ascending aorta. Patient will present with fever, chills,
petechial skin lesions, and splinter hemorrhages. A CT scan is
diagnostic. The above patient does not present with any evidence of a
prior infection.

Educational Objective:
Descending aortic aneurysm in a young male is usually due to blunt
trauma to the chest.

__________________________________________________________ 101
USMLE WORLD STEP 2 CK (INTERNAL MEDICINE) *Block
2*
Q NO 87: A 6 year-old-girl with hereditary spherocytosis is scheduled for a
splenectomy. She was previously managed with folate therapy and
occasional blood transfusions, but her anemia became refractory to medical
management alone. Before the operation, she is told that she will have an
enhanced risk of developing pneumococcal sepsis. She then asks, How long
will this risk last? What is the best response to her question?

A. 2 weeks
B. Up to 6 months
C. Up to 2 years
D. Up to 10 years
E. More than 10 years

Explanation:
Hereditary spherocytosis is an autosomal dominant disorder. It is
characterized by a lack of spectrin in the red cell membrane, which
causes the cells to become spheres, instead of being normal, flexible
and durable biconcave discs. The poorly flexible spherical cells are
thus unable to pass through the small fenestrations in the splenic red
pulp, and hemolysis takes place when the red cells are trapped within
the spleen.
The treatment for most patients involves supportive care with oral
folic acid and blood transfusions during periods of extreme anemia.
Splenectomy is considered if patients have moderate to severe
spherocytosis, or are refractory to medical management. The benefits of
splenectomy must be balanced against the immediate and long-term risks
of the procedure. Life-threatening anemia and the need for regular
transfusions maybe abolished by splenectomy, although a mild degree of
anemia usually persists. Immediate risks (e.g., hemorrhage,
postoperative infection, injury to nearby organs) are infrequent. The
most feared long-term complication is overwhelming sepsis with
encapsulated bacteria, most commonly Streptococcus pneumoniae.
Studies have shown that the risk for pneumococcal sepsis is present up
to 30 years and probably longer after splenectomy. To decrease this
risk, current recommendations call for the administration of anti-
pneumococcal, Haemophilus, and meningococcal vaccines several weeks
before the operation, and daily oral penicillin prophylaxis for three
to five years following splenectomy or until adulthood (for pediatric
patients). In view of reported deaths from sepsis up to 30 years or
more after splenectomy, a case can be made for lifetime penicillin
prophylaxis. Alternatively. antibiotics can be made available at home
for immediate treatment of any significant fever.

Educational Objective:
Studies have shown that the risk for sepsis is present up to 30 years
and probably longer alter splenectomy. Current recommendations state
that patients should receive anti-pneumococcal, Haemophilus, and
meningococcal vaccines several weeks before the operation, and daily
oral penicillin prophylaxis for three to five years following
splenectomy.
Extremely important question for the USMLE step-2

102 __________________________________________________________
USMLE WORLD STEP 2 CK (INTERNAL MEDICINE) *Block
2*
Q NO 88: A 59-year-old man comes to the physician because of dysuria,
urinary urgency, and frequency. He also complains of hematuria,
nocturia, decreased force of stream, and a feeling of incomplete voiding.
These symptoms have been present for more than a month. For the past
few days he has been experiencing dull, non-radiating suprapubic pain.
He has also been having low-grade fever and malaise for the past couple
of days. He has been taking analgesics for osteoarthritis of his right knee
for more than 10 years. He has a 40-pack-years history of cigarette
smoking and does not use alcohol or drugs. Rectal examination shows a
smooth, firm enlargement of the prostate without induration or
asymmetry. Neurological examination shows no abnormalities. Urinalysis
shows hematuria with isomorphic red blood cells. Laboratory studies
show a serum creatinine of 5 mg/dL, and a PSA of 2 ng/mL. Which of the
following is the most likely explanation for this patients symptoms?

A. Benign prostatic hyperplasia


B. Carcinoma of bladder
C. Carcinoma of prostate
D. Urethral stricture
E. Neurogenic bladder

Explanation:
The above patient has presented with irritative and obstructive voiding
symptoms. Such symptoms may be produced by multiple causes like benign
prostatic hyperplasia, bladder cancer, prostate cancer, urethral
stricture, neurogenic bladder and urinary tract infections. Therefore,
in all such patients, careful history, examination and certain lab
tests should be performed to find out the exact cause of voiding
complaints.
Benign prostatic hyperplasia can produce both irritative and
obstructive voiding symptoms, but presence of benign prostatic
hyperplasia is poorly correlated with voiding symptoms. Presence of a
firm, enlarged prostate on rectal examination does not always mean that
voiding symptoms are due to an enlarged prostate as most patients with
prostate enlargement are asymptomatic. Therefore, all the other causes
should be ruled out before attributing symptoms to benign prostatic
hyperplasia. Patients with benign prostatic hyperplasia usually do not
have hematuria, suprapubic pain and systemic features (Choice A).
Presence of hematuria with irritative or voiding symptoms should alarm
the physician to the possibility of bladder cancer. Bladder stones may
also produce hematuria. Presence of suprapubic pain may also occur with
bladder cancer. If bladder cancer is advanced, it may produce
suprapubic pain due to involvement of perivesical nerves or due to
obstruction of bladder outlet and urinary retention. Systemic features
appear when bladder cancer is advanced or metastatic. This patient has
a long hi story of analgesic use that is a risk factor for the
development of bladder cancer. Cigarette smoking is another risk
factor. Based on all these findings, this patient is most likely
suffering from bladder cancer (Choice B).
(Choice C) Patients with prostate cancer have induration of their gland
on rectal examination and PSA is greater than 4 ng/ml in such patients.

__________________________________________________________ 103
USMLE WORLD STEP 2 CK (INTERNAL MEDICINE) *Block
2*
(Choice D) Patients with urethral stricture usually give a history of
urethral trauma or urethral instrumentation. Hematuria is unlikely in
stricture.
(Choice E) Patients with neurogenic bladder have abnormal neurological
examination. There may be a history of neurological disease, diabetes,
stroke or back injury.

Educational Objective:
Presence of hematuria in a patient with irritative or obstructive
voiding should alert the physician to the possibility of bladder
cancer.

104 __________________________________________________________
USMLE WORLD STEP 2 CK (INTERNAL MEDICINE) *Block
2*
Q NO 89: A 65-year-old bedridden woman is brought in with complaints of
weight loss weakness and malaise. Her past medical history includes
chronic obstructive pulmonary disease (diagnosed fifteen years ago) and
hypertension often years duration. She quit smoking two years ago, but
previously smoked three packs of cigarettes daily since she was 20 years of
age. Her vital signs are stable. Her physical examination reveals severe
weakness in her proximal muscles, and loss of deep tendon reflexes. Chest
x-ray shows a right upper lung mass with mediastinal lymphadenopathy.
Which of the following is the most likely cause of her weakness?

A. Autoantibodies against post synaptic receptors


B. Immune mediated muscle inflammation
C. Upper and lower motor neuron degeneration
D. Multicentric CNS inflammation and demyelination
E. Antibodies to voltage gaited calcium channels

Explanation:
This patients history (i.e., heavy smoking, weight loss) and physical
findings (i.e., proximal muscle weakness, malaise, lung mass) are
suggestive of Myasthenic or Lambert-Eaton syndrome, which can occur in
association with small cell carcinoma of the lung. Lambert-Eaton
syndrome is caused by autoantibodies that are directed against the
voltage-gated calcium channels in the presynaptic motor nerve terminal.
This leads to the defective release of acetylcholine, thereby leading
to proximal muscle weakness. Electrophysiological studies confirm the
diagnosis (the muscle response to motor nerve stimulation should
increase with repetitive stimulation). Treatment consists of
plasmapheresis and immunosuppressive drug therapy.
(Choice A) Autoantibodies against postsynaptic receptors cause
myasthenia gravis. Reduction of postsynaptic acetylcholine receptors
leads to muscle weakness. The muscle weakness is provoked by repetitive
or sustained use of the muscles involved, unlike myasthenic syndrome.
Deep tendon reflexes are usually preserved, and may be somewhat brisk
in clinically weak muscles.
(Choice B) Immune-mediated muscle inflammation suggests polymyositis.
It presents as proximal weakness characterized by difficulty ascending
and descending stairs, combing hair, kneeling down, etc. It is not
associated with absent reflexes.
(Choice C) Upper and lower motor neuron degeneration suggests
amyotrophic lateral sclerosis. It initially presents with bulbar
symptoms, followed by upper and lower limb weakness.
(Choice D) Multicentric CNS inflammation and demyelination are
indicative of multiple sclerosis (MS). MS is rare in people younger
than ,5 and older than 60 years of age. The classic presentation
includes sensory loss, spasticity, cerebellar symptoms, optic neuritis,
depression and many other neuropsychological dysfunctions.

Educational Objective:
Eaton-Lambert syndrome is associated with small cell carcinoma of the
lung, and results from autoantibodies directed against the voltage-
gated calcium channels in the presynaptic motor nerve terminal.

__________________________________________________________ 105
USMLE WORLD STEP 2 CK (INTERNAL MEDICINE) *Block
2*
Q NO 90: A case-control study was conducted to assess the relationship
between tampon use and toxic shock syndrome (TSS). The odds ratio for
tampon use comparing the patients with TSS and the healthy subjects
(controls) was 5.0 (p <0.001). The authors concluded that the risk of
TSS is 5times higher in tampon users. The conclusion is valid if which of
the following assumptions is satisfied?

A. The sample size is big


B. The odds ratio is highly statistically significant
C. The confidence interval for the odds ratio is tight
D. TSS is a rare disease
E. The exposure is not associated with the disease

Explanation:
Case-control studies are very popular in exploring an exposure-disease
association, because they are relatively cheap and less time-consuming
than cohort studies. One of the major drawbacks of case-control studies
is the fact that the risk cannot be derived directly from their
results.
Exposure-odds ratio is the measure of association in case-control
studies. It compares the odds of exposure in cases to the odds of
exposure in controls. Itis NOT the same as relative risk. Relative risk
can be calculated in follow-up studies by comparing the risk in exposed
individuals to the risk in unexposed individuals. Direct calculation of
the relative risk is not possible in case-control studies, because the
study design does not include following people overtime. Nevertheless,
the relative risk can sometimes be approximately equal to the odds
ratio. If the prevalence of the disease is low exposure, the odds ratio
approximates the relative risk. This statement is called the rare
disease assumption and represents one of the fundamental epidemiologic
concepts.
(Choices A, B and C) Increasing the sample size would decrease the p
value for the odds ratio and make the confidence interval tighter. The
precision of the estimate would increase; but, that fact does not
affect the approximation of the odds ratio to the relative risk.
(Choice E)The results of the study suggest that the exposure is
associated with the disease.

Educational Objective:
If the outcome of a case-control study is not common in the population,
the odds ratio is close to the relative risk.

106 __________________________________________________________
USMLE WORLD STEP 2 CK (INTERNAL MEDICINE) *Block
2*
Q NO 91: A 28-year-old man with a history of intravenous drug abuse
presents with high-grade fever, rigors, and chills for the last two days.
He has no histor of pre-existing heart disease, but was admitted to
the hospital six months ago for cellulitis of the arm. His temperature is
40.0 C (104 F), pulse is 110/min, respirations are 22/min, and blood
pressure is 110/65 min Hg. Oropharynx is clear. Lungs are clear to
auscultation. A new holosystolic murmur is heard at the left second
intercostal space and along the left sternal border. His blood is drawn
and sent for culture. What is the most appropriate empiric antibiotic
therapy for this patient?
A. Vancomycin and gentamicin
B. Nafcillin and gentamicin
C. Cefazolin and gentamicin
D. Nafcillin and cefepime
E. Penicillin and clindamycin

Explanation:
Any IV drug abuser with fever and new-onset murmur should be deemed to
have infective endocarditis (IE) until proven otherwise. IV drug abuse
is a risk factor for IE, and the usual causative organism in this
situation is Staphylococcus aureus. The most appropriate empiric
antibiotic regimen in such a situation is vancomycin + gentamycin,
because this covers Staphylococci , Enterococci and Streptococci.
Furthermore, beta-lactam antibiotics are synergistic with
aminoglycosides. The other antibiotic choices do not cover all the
above-mentioned organisms.
(Choices B, C, D, and E) If the patient were not an IV drug abuser,
then gentamicin plus either nafcillin or cefazolin would have been a
reasonable choice. Methicillin-resistant Staphylococcus aureus (MRSA)
infections are on the rise. Thus, empiric antibiotic therapy should
cover MRSA. Penicillin is not a good choice because more than 99% of
Staphylococcus aureus strains are penicillin-resistant. Clindamycin can
cover some MRSA infections but should not be used for endocarditis.
Skin and soft tissue infections can be treated with clindamycin alone.

Educational Objective:
Infective endocarditis is especially common in IV drug abusers. It can
be fatal if initial empiric treatment (i.e., vancomycin + gentamicin
for IV drug abusers) is delayed.

__________________________________________________________ 107
USMLE WORLD STEP 2 CK (INTERNAL MEDICINE) *Block
2*
Q NO 92: An 8-month-old pale child is referred by a nurse practitioner
due to pale mucous membranes, irritability, and listlessness. The
stool examination is negative for occult blood, ova and parasites.
Laboratory studies reveal:
Hemoglobin 6.0 g/L
MCHC 25%
MCH 16.5 pg
MCV 68 fl
Reticulocytes 0.6 %
Platelets 230,000/mm
Leukocyte 5,500/mm
Neutrophils 56 %
Eosinophils 1 %
Lymphocytes 33 %
Monocytes 10 %
Serum Iron 40 mg/dL
TIBC 460 mg/dL (normal 300-350 mg/dL)
Percent saturation of transferrin 8.7 %
Total serum bilirubin 0.9 mg/dL
The peripheral blood smear shows marked anisocytosis, microcytosis,
hypochromia, and poikilocytosis. Which of the following is the most
likely diagnosis?

A. Iron deficiency anemia


B. Sideroblastic anemia
C. Dimorphic anemia
D. Megaloblastic anemia
E. Anemia of chronic disease

Explanation:
This patient has a microq4ic anemia (RBC indices: decreased MCV and
MCH), and the findings on the peripheral blood film (i.e., hypochromic,
microcytic erythrocytes) are suggestive of iron deficiency anemia. The
characteristic laboratory findings of iron deficiency anemia are
decreased serum iron level, decreased percent saturation (serum iron
/TIBC), and increased TIBC. The most common cause is inadequate dietary
intake of iron. The typical setting involves small children with a diet
consisting largely of milk, and without supplementation of iron-
fortified food products during the early years of development. Other
causes of iron deficiency anemia are bleeding and parasitic infections.
Hemolysis can also cause microcytic anemia (i.e., PNH); however, the
serum bilirubin level is typically elevated in such patients.
(Choice B) Sideroblastic anemia is characterized by increased serum
iron levels and normal TIBC.
(Choice C) In dimorphic anemia, two distinct forms of red cells are
circulating.
(Choice D) Megaloblastic anemia is characterized by an elevated MCV,
elevated MCH, and normal MCHC.
(Choice E) Anemia of chronic disease is associated with decreased TIBC.

Educational Objective:
The characteristic laboratory findings of iron deficiency anemia are
decreased serum iron level, decreased percent saturation (serum iron
/TIBC), and increased TIBC.

108 __________________________________________________________

You might also like